vision ias - wordpress.com · 2018-11-05 · 3 ©vision ias service delivery preparedness and...

39
1 www.visionias.in ©Vision IAS VISIONIAS www.visionias.in ANSWERS & EXPLANATION GENERAL STUIDES (P) TEST 2661 (2019) Q 1.D o The Consumer Protection Act, 1986 provides for a system for the protection of consumer rights and for the redressal of consumer disputes. The consumer rights provided in the Act are: Right to Safety: Means right to be protected against the marketing of goods and services, which are hazardous to life and property. The purchased goods and services availed of should not only meet their immediate needs but also fulfill long-term interests. Before purchasing, consumers should insist on the quality of the products as well as on the guarantee of the products and services. They should preferably purchase quality marked products such as ISI, AGMARK, etc Right to be Informed: Means right to be informed about the quality, quantity, potency, purity, standard and price of goods so as to protect the consumer against unfair trade practices. The consumer should insist on getting all the information about the product or service before making a choice or a decision. This will enable him to act wisely and responsibly and also enable him to desist from falling prey to high-pressure selling techniques. Right to Choose: Means right to be assured, wherever possible of access to a variety of goods and services at a competitive price. In the case of monopolies, it means right to be assured of satisfactory quality and service at a fair price. It also includes the right to basic goods and services Right to be Heard: Means that consumer's interests will receive due consideration at appropriate forums. It also includes the right to be represented in various forums formed to consider the consumer's welfare. Consumers should form non-political and non-commercial consumer organizations which can be given representation in various committees formed by the Government and other bodies in matters relating to consumers. Right to seek Redressal: Means right to seek redressal against unfair trade practices or unscrupulous exploitation of consumers. It also includes the right to a fair settlement of the genuine grievances of the consumer. Consumers must make a complaint about their genuine grievances. Right to Consumer Education: Means the right to acquire the knowledge and skill to be an informed consumer throughout life. Ignorance of consumers, particularly of rural consumers, is mainly responsible for their exploitation. They should know their rights and must exercise them. Right to Clean and Healthy Environment Hence, all the statements are correct. Q 2.D o The Cabinet Secretariat is under the direct charge of the Prime Minister. The administrative head of the Secretariat is the Cabinet Secretary who is also the ex-officio Chairman of the Civil Services Board. In the Government of India (Allocation of Business) Rules, 1961 "Cabinet Secretariat" finds a place in the First Schedule to the Rules. The subjects allotted to this Secretariat are:- Secretarial assistance to Cabinet and Cabinet Committees. Rules of Business. https://telegram.me/UPSCMaterials https://telegram.me/FreeUPSCMaterials https://telegram.me/MaterialforExam

Upload: others

Post on 19-Mar-2020

6 views

Category:

Documents


0 download

TRANSCRIPT

Page 1: VISION IAS - WordPress.com · 2018-11-05 · 3 ©Vision IAS Service Delivery Preparedness and achievement of Results Sound Public Grievance Redress Mechanism o Sevottam has also been

1 www.visionias.in ©Vision IAS

VISIONIAS www.visionias.in

ANSWERS & EXPLANATION

GENERAL STUIDES (P) TEST – 2661 (2019)

Q 1.D

o The Consumer Protection Act, 1986 provides for a system for the protection of consumer rights and for

the redressal of consumer disputes. The consumer rights provided in the Act are:

Right to Safety: Means right to be protected against the marketing of goods and services, which

are hazardous to life and property. The purchased goods and services availed of should not only

meet their immediate needs but also fulfill long-term interests. Before purchasing, consumers should

insist on the quality of the products as well as on the guarantee of the products and services. They

should preferably purchase quality marked products such as ISI, AGMARK, etc

Right to be Informed: Means right to be informed about the quality, quantity, potency, purity,

standard and price of goods so as to protect the consumer against unfair trade practices. The consumer

should insist on getting all the information about the product or service before making a choice or a

decision. This will enable him to act wisely and responsibly and also enable him to desist from falling

prey to high-pressure selling techniques.

Right to Choose: Means right to be assured, wherever possible of access to a variety of goods and

services at a competitive price. In the case of monopolies, it means right to be assured of satisfactory

quality and service at a fair price. It also includes the right to basic goods and services

Right to be Heard: Means that consumer's interests will receive due consideration at

appropriate forums. It also includes the right to be represented in various forums formed to

consider the consumer's welfare. Consumers should form non-political and non-commercial

consumer organizations which can be given representation in various committees formed by the

Government and other bodies in matters relating to consumers.

Right to seek Redressal: Means right to seek redressal against unfair trade practices or unscrupulous

exploitation of consumers. It also includes the right to a fair settlement of the genuine grievances of

the consumer. Consumers must make a complaint about their genuine grievances.

Right to Consumer Education: Means the right to acquire the knowledge and skill to be an informed

consumer throughout life. Ignorance of consumers, particularly of rural consumers, is mainly

responsible for their exploitation. They should know their rights and must exercise them.

Right to Clean and Healthy Environment

Hence, all the statements are correct.

Q 2.D

o The Cabinet Secretariat is under the direct charge of the Prime Minister. The administrative head of the

Secretariat is the Cabinet Secretary who is also the ex-officio Chairman of the Civil Services Board. In the

Government of India (Allocation of Business) Rules, 1961 "Cabinet Secretariat" finds a place in the First

Schedule to the Rules. The subjects allotted to this Secretariat are:-

Secretarial assistance to Cabinet and Cabinet Committees.

Rules of Business.

https://telegram.me/UPSCMaterials https://telegram.me/FreeUPSCMaterials https://telegram.me/MaterialforExam

Page 2: VISION IAS - WordPress.com · 2018-11-05 · 3 ©Vision IAS Service Delivery Preparedness and achievement of Results Sound Public Grievance Redress Mechanism o Sevottam has also been

2 www.visionias.in ©Vision IAS

o The Secretariat assists in decision-making in Government by ensuring Inter-Ministerial coordination,

ironing out differences amongst Ministries/Departments and evolving consensus through the

instrumentality of the standing/ad-hoc Committees of Secretaries.

o The Directorate of Public Grievances was set up in the Cabinet Secretariat in March 1988. This

Directorate entertains grievances from the public. Citizens can file grievances, either on paper or online

with the Directorate of Public Grievance in respect of Departments/organizations which have an extensive

public interface such as MTNL/BSNL, Railways, Posts, Insurance Companies, Public Sector Bank, etc.

Depending on nature and gravity of the grievances, the Directorate either seeks comments or transfers the

same for appropriate action to the concerned Department(s).

o National Authority, Chemical Weapons Convention (CWC) was set up by a resolution of Cabinet

Secretariat dated 5th May 1997 to fulfill the obligations enunciated in the Chemical Weapons

Convention initially signed by 130 countries in a conference which concluded on 14th January 1993 for

the purpose prohibiting of the development, production, execution, transfer, use and stockpiling of all

chemical weapons by Member-States is a non-discriminatory process. To fulfill its obligations, each State

Party has to designate or establish a National Authority to serve as the national focal point for effective

liaison with Organization for Prohibition of the Chemical Weapons and other State Parties and hence the

NA, CWC under the administrative control of the Cabinet Secretariat was set up. The National Authority

is headed by the Chairperson who is in the rank of Additional Secretary to the Government of India and is

supported by a suitable Technical Secretariat to look after the various functions.

o Direct Benefit Transfer(DBT) Mission is a major reform initiative where benefits, cash or in-kind, are

delivered directly to identified beneficiaries using Aadhaar. It envisages efficiency and inclusion in the

delivery processes leading to greater accountability and transparency in the system. DBT Mission was

created in the Planning Commission to act as a nodal point for implementation of the DBT. The Mission

was transferred to the Department of Expenditure in July‟13 and shifted to Cabinet Secretariat

w.e.f. 14.09.2015.

Q 3.D

o The National Commission for SCs consists of a chairperson, a vice-chairperson and three other

members. They are appointed by the President by warrant under his hand and seal.

o Their conditions of service and tenure of office are also determined by the President.

o Hence, both statements are not correct.

o Constitution of India under Article 338 has assigned the following duties and functions to the

Commission.

To investigate and monitor all matters relating to the safeguards provided for the Scheduled Castes

under the Constitution or under any other law for the time being in force or under any order of the

Government and to evaluate the working of such safeguards;

to inquire into specific complaints with respect to the deprivation of rights and safeguards of the

Scheduled Castes.

Q 4.C

o Sevottam Service Delivery Excellence Model

o Sevottam is an assessment - improvement model that has been developed with the objective of

improving the quality of public service delivery in the country. The model was conceived by

the Department of Administrative Reforms & Public Grievances (DARPG), Ministry of Personnel,

Public Grievances, and Pensions in 2006. Hence statement 1 is correct.

o The word "Sevottam" is a combination of two Hindi words: Seva (Service) and Uttam (Excellent). It

means ―Service Excellence‖, emphasizing the idea of ―Service‖. It symbolizes the change in mindset

within the Government, from administration and control to service and enablement.

o The key components of Sevottam are captured with the following objectives:

Successful implementation of Citizen‟s Charters

https://telegram.me/UPSCMaterials https://telegram.me/FreeUPSCMaterials https://telegram.me/MaterialforExam

Page 3: VISION IAS - WordPress.com · 2018-11-05 · 3 ©Vision IAS Service Delivery Preparedness and achievement of Results Sound Public Grievance Redress Mechanism o Sevottam has also been

3 www.visionias.in ©Vision IAS

Service Delivery Preparedness and achievement of Results

Sound Public Grievance Redress Mechanism

o Sevottam has also been launched as a certification scheme which provides for the award of the Sevottam

symbol of excellence to public service organizations that implement and are able to show compliance to a

set of management system requirements that have been specified in a specially created standard

document. This standard, known as IS 15700:2005, was developed by the Bureau of Indian Standards

(BIS) based on the objectives of Sevottam. Hence statement 2 is correct.

o The standard takes into account unique conditions of service delivery by Public service organizations in

India and the sectoral and regional variations in service delivery standards. It offers a systematic way to

identify weaknesses in specific areas and rectify them through systemic changes and process re-

engineering.

o India is among the first countries in the world to have a Quality Standard for public service delivery.

Q 5.D

o Statement 1 is not correct: We currently live in what is called the Holocene Epoch, which reflects

everything that has happened over the past 11,700 years - since a dramatic warming kicked us out of the

last ice age. Meghalayan Age is a part of Holocene Series/Epoch. Further, the Holocene is subdivided into

three parts, the Greenlandian, the Northgrippian, and the youngest addition, the Meghalayan. The

Greenlandian runs from 11,700 to 8,200 years ago; the Northgrippan runs from 8,200 to 4,200 years ago,

and finally, the Meghalayan runs from 4,200 years ago to present. They are calling it the Meghalayan

Age, the onset of which was marked by a mega-drought that crushed a number of civilisations worldwide.

o Pleistocene is the period in Earth's history which is commonly referred to as the Ice Age. The Pleistocene

started 2.6 million years ago and lasted until the termination of the Weichsel glaciation about 11,711 years

ago.

o Statement 2 is correct: A stalagmite which was found in Mawmluh cave formation in the state of

Meghalaya has provided chemical signatures as evidence. The stalagmite is a type of rock formation that

forms on the floor of a cave due to the accumulation from ceiling drippings.

o Statement 3 is not correct: It is the International Commission on Stratigraphy and not WMO which

is the official timekeeper of geological time and has declared thereby this new age. The International

Commission on Stratigraphy is the largest and oldest constituent scientific body in the International

Union of Geological Sciences (IUGS). Its primary objective is to precisely define global units (systems,

series, and stages) of the International Chronostratigraphic Chart that, in turn, are the basis for the units

(periods, epochs, and age) of the International Geologic Time Scale, thus setting global standards for the

fundamental scale for expressing the history of the Earth.

o The World Meteorological Organization (WMO) is an intergovernmental organization with a

membership of 191 Member States and Territories.

It originated from the International Meteorological Organization (IMO), which was founded in 1873.

Established by the ratification of the WMO Convention on 23 March 1950, WMO became the

specialised agency of the United Nations for meteorology (weather and climate), operational

hydrology and related geophysical sciences a year later. The Secretariat, headquartered in Geneva, is

headed by the Secretary-General.

Its supreme body is the World Meteorological Congress.

WMO provides world leadership and expertise in international cooperation in the delivery and use of

high-quality, authoritative weather, climate, hydrological and related environmental services by its

Members, for the improvement of the well-being of societies of all nations.

Q 6.D

o In the scheme of parliamentary system of government provided by the Indian Constitution, the President

is the nominal executive authority and Prime Minister is the real executive authority. Prime Minister

heads a series of departments/Committees/institutions/ministries. Some of the major ones include-

https://telegram.me/UPSCMaterials https://telegram.me/FreeUPSCMaterials https://telegram.me/MaterialforExam

Page 4: VISION IAS - WordPress.com · 2018-11-05 · 3 ©Vision IAS Service Delivery Preparedness and achievement of Results Sound Public Grievance Redress Mechanism o Sevottam has also been

4 www.visionias.in ©Vision IAS

NITI Aayog

National Ganga River Basin Authority (NGRBA)

National Development Council (NDC)

Department of Science (DoS)

National Commission for Population Control

Department of Atomic Energy

Nuclear Command Authority

Indian Board of Wildlife

National Disaster Management Authority

Council of Scientific and Industrial Research (CSIR)

PM‘s Council on Climate Change

Inter-state Council

Appointments Committee to the Cabinet

Cabinet Committee on Economic Affairs

Cabinet Committee on Political Affairs

Ministry of Personnel, Public Grievances

Lokpal Selection Committee

CBI Director Search Committee

CVC Selection Committee

Hence, option (d) is the correct answer.

Q 7.C

o Recently, India has become the vice-chair (regional head) of the Asia Pacific region of the World

Customs Organization (WCO) for a period of two years till June 2020. Ministry of Finance is the nodal

agency representing India in WCO. Hence statement 3 is not correct.

o The WCO has divided its Membership into six Regions. Each of the six Regions is represented by a

regionally elected Vice-Chairperson to the WCO Council.

o The World Customs Organization (WCO), established in 1952 as the Customs Co-operation Council

(CCC) is an independent intergovernmental body whose mission is to enhance the effectiveness and

efficiency of Customs administrations. Hence statement 1 is not correct and 2 is correct.

o The WCO represents 182 Customs administrations across the globe that collectively process

approximately 98% of world trade. As the global center of Customs expertise, the WCO is the only

international organization with competence in Customs matters and can rightly call itself the voice of the

international Customs community.

Q 8.A

o Anybody that derives its power from a 'Law' or 'Statute' made by Parliament is called as a statutory

body or statutory authority. For example:

The Competition Commission of India (CCI) was established under the Competition Act, 2002 for

the administration, implementation and enforcement of the Act, and was duly constituted in March

2009. The following are the objectives of the Commission.

o To prevent practices having an adverse effect on competition.

o To promote and sustain competition in markets.

o To protect the interests of consumers and

o To ensure freedom of trade

o The University Grants Commission (UGC) came into existence on 28th December 1953 and became a

statutory organization of the Government of India by an Act of Parliament in 1956. The UGC has the

unique distinction of being the only grant-giving agency in the country which has been vested with two

responsibilities: that of providing funds and that of coordination, determination, and maintenance of

standards in institutions of higher education.

https://telegram.me/UPSCMaterials https://telegram.me/FreeUPSCMaterials https://telegram.me/MaterialforExam

Page 5: VISION IAS - WordPress.com · 2018-11-05 · 3 ©Vision IAS Service Delivery Preparedness and achievement of Results Sound Public Grievance Redress Mechanism o Sevottam has also been

5 www.visionias.in ©Vision IAS

o Law Commission of India (LCI) is neither a constitutional nor a statutory body. The first LCI was

established in 1834 under the Charter Act of 1833 under the Chairmanship of Lord Macaulay. The first

Law Commission of independent India was established in 1955 for a three-year term. LCI submitted its

report to the Ministry of Law and Justice. Law Commission of India is established by an order of central

government. Who will head the law commission is completely at the discretion of the

Government. However, it is a convention that a retired judge of Supreme Court heads India‘s Law

Commission. Further, the States also can constitute their own law commissions.

o The National Commission for Minority Educational Institutions was set up in 2004. The Government

brought out an Ordinance in November 2004 establishing the Commission. Later a Bill was introduced

in the Parliament in December 2004 and both Houses passed the Bill. The Commission is mandated to

look into specific complaints regarding deprivation or violation of rights of minorities to establish and

administer educational institutions of their choice. The Commission is a quasi-judicial body and has been

endowed with the powers of a Civil Court. It is to be headed by a Chairman who has been a Judge of the

High Court and three members are to be nominated by Central Government.

Q 9.B

o Model Concession Agreement (MCA) forms the core of the public-private partnership (PPP)

projects in India. Hence option (b) is the correct answer.

o The MCA spells out the policy and regulatory framework for implementation of a PPP project.

o It addresses a gamut of critical issues pertaining to a PPP framework like:

Mitigation and unbundling of risks

Allocation of risks and returns

The symmetry of obligations between the principal parties

Precision and predictability of costs & obligations

Reduction of transaction costs and termination.

o The MCA allocates risk to parties best suited to manage them. The Model Concession Agreements for

various sectors like National Highways, State Highways, Urban Rail Transit System and Ports are

available. Planning Commission developed the first version of the Model Concession Agreement (MCA).

This was done considering the need to standardize documents and processes for the PPP framework in the

country for ensuring uniformity, transparency, and quality in the development of large-scale infrastructure

projects.

o Subsequently, the Planning Commission developed various other versions of the MCA considering the

different PPP modes like Built Operate Transfer (BOT) (Toll), BOT (Annuity), Design, Build, Operate

and Transfer (DBOT) and Operate Maintain and Transfer (OMT) addressing to a significant extent, the

changing needs of the sector.

Q 10.D

o Statement 1 is not correct: The ‗two plus two dialogue‘ is an expression used to indicate that two

appointed ministers from each country, the ministers of defence and external affairs, in this case, will

meet up to discuss the two countries‟ strategic and security interests. The goal is to establish a

diplomatic, yet fruitful, conversation between the two countries‘ respective heads of defence and external

affairs.

o Statement 2 is correct: India and the US have conducted their inaugural meeting on September

6, which was announced by the President of United States and the Prime Minister of India, during his visit

to the US last year.

o Statement 3 is not correct: India has a 2+2 dialogue with Japan as well. Japan leans on this

mechanism for its interactions with the US, France, Russia and Australia. India, on the other hand, has

established the 2+2 dialogue primarily with Japan. Both the countries have practised a ‗two plus two

dialogue‘ mechanism almost annually, held considerably before the meeting of the two prime ministers,

since 2010. This meeting between the external affairs and defence ministers of two countries helps amass

the issues both the prime ministers need to tackle later in their scheduled meetings.

https://telegram.me/UPSCMaterials https://telegram.me/FreeUPSCMaterials https://telegram.me/MaterialforExam

Page 6: VISION IAS - WordPress.com · 2018-11-05 · 3 ©Vision IAS Service Delivery Preparedness and achievement of Results Sound Public Grievance Redress Mechanism o Sevottam has also been

6 www.visionias.in ©Vision IAS

Q 11.D

o India granted voting rights to transgenders in 1994. The Sixty-first Amendment of the Constitution of

India, officially known as the Constitution (Sixty-first Amendment) Act, 1988, lowered the voting age of

elections to the Lok Sabha Legislative Assemblies of States from 21 years to 18 years. Hence statement

1 is not correct.

o Shabnam Mausi Bano was elected India‘s first transgender Member of the Legislative Assembly (MLA)

in 1999 from Madhya Pradesh‘s Sohagpur constituency.

o In a landmark decision in National Legal Services Authority vs Union of India case 2014, the Supreme

Court of India recognized the transgender as the third gender. It directed the government to treat

transgender persons as ―socially and educationally backward classes and extend all kinds of reservation‖

in government jobs and higher education to them under the OBC quota.

o However, the above direction is not yet fulfilled by the government. Hence statement 2 is not correct.

o The Transgender Persons (Protection of Rights) Bill, 2016 is yet to be enacted. Moreover, it is not yet

confirmed that whether the bill will provide for reservation for the transgender community or not.

Q 12.C

o The Consumer Guidance Society of India was formed by nine ladies to organize a movement to fight for

consumer rights and resist consumer exploitation. CGSI is the first consumer organization in India,

founded in 1966. It was the first to demand a Consumer Protection Act with Consumer Courts to

implement it. This became a reality in 1986. Hence, statement 1 is correct.

o CGSI promotes consumer education; initiates training projects in rural areas; promotes publicity drives;

and represents consumer interests with Government and other bodies. Hence, statement 2 is correct.

Q 13.C

o In our country, MSP for 24 agricultural commodities of Kharif and Rabi season is announced by the

Government based on the recommendations of the Commission for Agriculture Cost and Prices

(CACP). However, procurement by Central and State Agencies is limited to rice and wheat and some

amount of coarse cereals. The Government also procures a limited quantity of oilseed and pulses through

NAFED, SFAC, and some other agencies. The Market intervention scheme (MIS) is implemented in

case of the prices falling below the threshold level in perishable crops.

o Three concepts are being opinioned by NITI AYOG.

The first option related to Market Assurance Scheme, which proposes procurement by States and

compensation of losses to the certain extent of MSP after the procurement and price realization out of

the sale of the procured produce.

Second option related to price deficiency procurement scheme. Under this scheme, if the sale price

is below a modal price then the farmers may be compensated to the difference between MSP and

actual price subject to a ceiling which may not exceed 25% of the MSP. No compensation would be

due if the modal price in neighboring States is above the MSP.

Third option related to Private Procurement and Stockist Scheme, which relates to procurement by

private entrepreneurs at MSP and Government providing some policy and tax incentives and a

commission to such private entities which may be decided on the basis of transparent criteria and

bidding for the empanelment of private players by the State Government to do the procurement

operations.

o Commission for Agricultural Costs and Prices (CACP) has suggested that the Centre explore the

possibility of implementing the price deficiency payment (PDP) scheme across the country. Madhya

Pradesh had implemented such a scheme — the Bhavantar Bhugtan Yojana (BBY) — on a pilot basis

during last year‘s kharif season to safeguard farmers from price fluctuations.

https://telegram.me/UPSCMaterials https://telegram.me/FreeUPSCMaterials https://telegram.me/MaterialforExam

Page 7: VISION IAS - WordPress.com · 2018-11-05 · 3 ©Vision IAS Service Delivery Preparedness and achievement of Results Sound Public Grievance Redress Mechanism o Sevottam has also been

7 www.visionias.in ©Vision IAS

Q 14.D

o Parliament repealed the Persons with Disabilities (Equal Opportunities, Protection of Rights and Full

Participation) Act,1995 and brought in the Rights of Persons with Disabilities Act, 2016.

o The 2016 Act recognises 21 kinds of disabilities compared to the previous seven which include:

Blindness

Low-vision

Leprosy Cured persons

Hearing Impairment (deaf and hard of hearing)

Locomotor Disability

Dwarfism

Intellectual Disability

Mental Illness

Autism Spectrum Disorder

Cerebral Palsy

Muscular Dystrophy

Chronic Neurological conditions

Specific Learning Disabilities

Multiple Sclerosis

Speech and Language disability

Thalassemia

Hemophilia

Sickle Cell disease

Multiple Disabilities including deafblindness

Acid Attack victim

Parkinson's disease

Q 15.D

o Golden Hand-Shake: In 1988, the government initiated the Voluntary Retirement Scheme (VRS) to

help the public enterprises to shed excess manpower. This scheme came to be popularly known as

“Golden Hand-Shake” Policy as the workers get a handsome amount from the enterprises at the time of

dissociation.

o In 1992, the National Renewal Fund (NRE) was created for training and redeployment of retrenched

workers, besides providing compensation to the persons seeking voluntary retirement. The National

Renewal Fund was abolished in 2000.

o Hence option (d) is the correct answer.

Q 16.A

o Social audit is a tool which measures, evaluates, identifies gaps in service delivery and elicits promises to

rectify these gaps with the direct participation of intended beneficiaries. In its essence, it empowers

citizens to gauge the effectiveness of a scheme by looking at its overall impact.

o It is an accountability tool that measures, evaluates, identifies gaps in service delivery and elicits promises

to rectify these gaps with the direct participation of intended beneficiaries in this process.

o It helps to empower citizens to be directly involved in the programmes of a welfare state and to raise

grievances before an authority , instead of being passive beneficiaries. A realisation of what one is entitled

to by law and the empowerment that comes along with demanding this rightful entitlement is a by-product

of a social audit.

o Hence, option (a) is the correct answer.

o Citizens Charter

It is a document which represents a systematic effort to focus on the commitment of the Organisation

towards its Citizens in respects of Standard of Services, Information, Choice and Consultation, Non-

https://telegram.me/UPSCMaterials https://telegram.me/FreeUPSCMaterials https://telegram.me/MaterialforExam

Page 8: VISION IAS - WordPress.com · 2018-11-05 · 3 ©Vision IAS Service Delivery Preparedness and achievement of Results Sound Public Grievance Redress Mechanism o Sevottam has also been

8 www.visionias.in ©Vision IAS

discrimination and Accessibility, Grievance Redress, Courtesy and Value for Money. This also

includes expectations of the Organisation from the Citizen for fulfilling the commitment of the

Organisation.

o Propriety Audit

It stands for verification of transactions on the tests of public interest commonly accepted customs

and standards of conduct. It is mostly conducted by a Specialized agency and doesn't involve the

direct participation of intended beneficiaries.

Q 17.D

o The Central Bureau of Investigation (CBI ) , functioning under Deptt. of Personnel, Ministry of Personnel,

Pension & Public Grievances, Government of India, is the premier investigating police agency in India.

o The Central Bureau of Investigation traces its origins to the Special Police Establishment (SPE)

established in 1941 by the government. The Central Bureau of Investigation (CBI ) , functioning under

Deptt. of Personnel, Ministry of Personnel, Pension & Public Grievances, Government of India, is the

premier investigating police agency in India.

o The functions of the SPE were to investigate bribery and corruption in transactions with the War and

Supply Department of India, set up during World War II with its headquarters in Lahore. The DSPE

acquired its popular current name, Central Bureau of Investigation (CBI), through a Home Ministry

resolution dated 1.4.1963.

o The CBI is headed by a director, an IPS officer with a rank of Inspector General of Police or

Commissioner of Police (State). The director is selected based on the CVC Act 2003 and has a two-year

term. Other ranks in the CBI which may be staffed by the Indian Revenue Service (IRS) and the IPS are

Special Director, Additional Director, Joint Director, Deputy Inspector General of Police, Senior

Superintendent of Police and Superintendent of Police.

o The superintendence of CBI related to investigation of offences under the Prevention of Corruption

Act, 1988 lies with the Central Vigilance Commission (CVC) and in other matters with the

Department of Personnel & Training (DOPT) in the Ministry of Personnel, Pension & Grievances

of the Government of India.

o The legal powers of investigation of the CBI are derived from the DSPE Act 1946, which confers

powers, duties, privileges and liabilities on the Delhi Special Police Establishment (CBI) and officers

of the Union Territories. The central government may extend to any area (except Union Territories) the

powers and jurisdiction of the CBI for investigation, subject to the consent of the government of the

concerned state. Members of the CBI at or above the rank of sub-inspector may be considered officers in

charge of police stations. Under the act, the CBI can investigate only with notification by the central

government.

Q 18.C

o The Delhi Dialogue, has emerged as India‟s foremost ASEAN-centred Track 1.5 Forum. The Delhi

Dialogue is a summit organized between India and ASEAN countries. No other SAARC nation is

involved in it. This year‘s theme is "Strengthening India-ASEAN Maritime Cooperation."

Q 19.C

o The Right of Children to Free and Compulsory Education Act or Right to Education Act (RTE), is an Act

of the Parliament of India enacted on 4 August 2009, which makes education a fundamental right under

Article 21A of the Indian Constitution for every child between the ages of 6 and 14 and specifies

minimum norms in elementary schools.

o It requires all private schools to reserve 25% of seats to children. But the provisions of RTE Act shall not

apply to Madrasas, Vedic Pathsalas and educational institutions primarily imparting religious

instruction. Hence statement 2 is correct.

https://telegram.me/UPSCMaterials https://telegram.me/FreeUPSCMaterials https://telegram.me/MaterialforExam

Page 9: VISION IAS - WordPress.com · 2018-11-05 · 3 ©Vision IAS Service Delivery Preparedness and achievement of Results Sound Public Grievance Redress Mechanism o Sevottam has also been

9 www.visionias.in ©Vision IAS

o The Act requires that central government shall ‗develop and enforce standards for training of

teachers‘. Hence statement 1 is correct.

o As per the Act child should be studied in the neighborhood school. Neighborhood school concept relates

to the availability of a school within a safe and accessible distance from the habitation where a child lives.

The Act defines the limits of neighborhood schools as 1 km walking distance from the habitation of a

child at the primary level (class 1 to 5) and within 3km walking distance for an upper primary level (class

6 to 8). Hence, statement 3 is not correct.

o Proof of age document would be required at the time of admission to determine which class she is to be

admitted since age-appropriate education is the provisions of the RTE. Birth certificate or such other

document issued by a statutory body may be issued for proof of age. However, if no such documents are

available with the parent, the child will still be admitted in the school. In such situations, the age of the

child as stated by the parents will be considered to determine the age-appropriate class for the

child. Hence statement 4 is correct.

Q 20.D

o The National Board for Wildlife (NBWL) recently added four species- the Northern River

Terrapin, Clouded Leopard, Arabian Sea Humpback Whale, Red Panda - to a Recovery

Programme for Critically Endangered Species. It is one of the components of centrally sponsored

scheme - Integrated Development of Wildlife Habitats ( IDWH ) which provides assistance to the

State/UT governments for activities aimed at wildlife.

About the Species –

o Northern River Terrapin, (IUCN-Critically Endangered), which is a species of riverine turtle found in

the rivers that flow in Eastern India, is hunted for its meat and carapace. It is a native of Bangladesh,

Cambodia, India, Indonesia, and Malaysia.

o Clouded Leopard, (IUCN-Vulnerable), found in the Himalayan foothills, is threatened due to habitat

loss, poaching for its skin and is also as a live pet trade.

o Arabian Sea Humpback, (IUCN-Endangered), it is a Whale a species found in all of the major oceans

but ship strikes, unforgiving fishing gear, and seismic explorations pose a grave threat to it.

o Red Panda, (IUCN-Endangered) which is closely associated with montane forests with dense bamboo-

thicket, is found Sikkim, West Bengal, and Arunachal Pradesh. It is poached for its meat, and for use in

medicines, and as a pet.

Q 21.B

o The government of India had notified the Electoral Bond Scheme 2018 in January 2018.

o As per provisions of the Scheme, Electoral Bonds may be purchased by a person, who is a citizen of India

or incorporated or established in India. A person being an individual can buy Electoral Bonds, either

singly or jointly with other individuals. Hence, statement 2 is not correct.

o Only the Political Parties registered under Section 29A of the Representation of the People Act, 1951 (43

of 1951) and which secured not less than one per cent of the votes polled in the last General Election to

the House of the People or the Legislative Assembly of the State, shall be eligible to receive the Electoral

Bonds. Hence, statement 1 is not correct.

o The Electoral Bonds can be issued in the denomination of Rs1000, Rs 10,000, Rs 1,00,000, Rs 10,00,000

and Rs 1,00,00,000 only.

o Currently, only SBI is authorized to sell these bonds. The bonds are valid only for 15 days from the date

of issue. The Electoral Bonds shall be encashed by an eligible Political Party only through a Bank account

with the Authorized Bank. Hence, statement 3 is correct.

https://telegram.me/UPSCMaterials https://telegram.me/FreeUPSCMaterials https://telegram.me/MaterialforExam

Page 10: VISION IAS - WordPress.com · 2018-11-05 · 3 ©Vision IAS Service Delivery Preparedness and achievement of Results Sound Public Grievance Redress Mechanism o Sevottam has also been

10 www.visionias.in ©Vision IAS

Q 22.B

o Recently, the National Commission for Protection of Child Rights (NCPCR) has issued its first-ever

National guidelines on regulating fees in private schools. It can also penalize the institution that violates

the norms created by it. The guidelines are titled as ―Model Fee Regulatory Mechanism‖.

o The Functions of the National Commission for Protection of Child Rights as laid out in the Commissions

for Protection of Child Rights (CPCR) Act, 2005 are as follows:

o Examine and review the safeguards provided by or under any law for the time being in force for the

protection of child rights and recommend measures for their effective implementation;

Present to the central government, annually and at such other intervals, as the commission may deem

fit, reports upon working of those safeguards;

Inquire into violation of child rights and recommend initiation of proceedings in such cases;

Examine all factors that inhibit the enjoyment of rights of children affected by terrorism, communal

violence, riots, natural disaster, domestic violence, HIV/AIDS, trafficking, maltreatment, torture and

exploitation, pornography and prostitution and recommend appropriate remedial measures;

Look into the matters relating to the children in need of special care and protection including children

in distress, marginalized and disadvantaged children, children in conflict with the law, juveniles

children without family and children of prisoners and recommend appropriate remedial

measures; NCPCR is also mandated by POCSO Act, 2012 and POCSO Rules, 2012.

Study treaties and other international instruments and undertake periodical review of existing policies,

programmes and other activities on child rights and make recommendations for their effective

implementation in the best interest of children;

Undertake and promote research in the field of child rights;

Spread child rights literacy among various section of society and promote awareness of the safeguards

available for protection of these rights through publications, the media, seminar, and other available

means;

Inspect or cause to be inspected any juveniles custodial home, or any other place of residence or

institution meant for children, under the control of the Central Government or any State

Government or any other authority, including any institution run by a social

organization; Where children are detained or lodged for the purpose of treatment, reformation or

protection and take up with these authorities for remedial action, if found necessary. Hence

statement 3 is correct.

Inquire into complaints and take suo motu notice of matter relating to :

Deprivation and violation of child rights. Hence statement 1 is correct.

Non-implementation of laws providing for protection and development of children;

Non-compliance of policy decisions, guidelines or instructions aimed at mitigating hardships to

and ensuring the welfare of the children and provide relief to such children; Or take up the issues

arising out of such matters with appropriate authorities.

o Such other functions as it may consider necessary for the promotion of Child Rights and any other matter

incidental to the above function.a state commission or any other commission duly constituted under any

law for the time being in force.

o The Commission shall not enquire into any matter which is pending before a State Commission or any

other Commission duly constituted under any law for the time being in force.

o Analyse existing law,policy and practice to assess compliance with Convention on the rights of the Child,

undertake inquiries and produce reports on any aspects of policy or practice affecting childrenand

comment on proposed new legislation related to child rights.

o Present to the Central Government annualy and at such other intervals as the Commission may deem

fit,reports upon the working of those safeguards.

o Undertake formal investigation where concern has been expressed either by children themselved or by

concerned person on their behalf.

https://telegram.me/UPSCMaterials https://telegram.me/FreeUPSCMaterials https://telegram.me/MaterialforExam

Page 11: VISION IAS - WordPress.com · 2018-11-05 · 3 ©Vision IAS Service Delivery Preparedness and achievement of Results Sound Public Grievance Redress Mechanism o Sevottam has also been

11 www.visionias.in ©Vision IAS

o Promote, respect and serious consideration of the views of children in its work and in that of all

Government Departments and Organisations dealing with Child.

o Produce and disseminate informtion about child rights.

o Compile and analyse data on children.

o Promote the incorporation of child rights into the school curriculum, training of teachers or

personnel dealing with children. Hence statement 2 is correct.

Q 23.B

o Behdienkhlam (chasing away the Demon of Cholera) is celebrated annually in July after the sowing

period, the most important dance festival of the Jaintia tribes. This festival is also an invocation to God,

seeking his blessings for a bumper harvest. The festival held at Jowai is one of the most well known

and recreational festival in Meghalaya.

o The women however do not participate in the dancing, as they have an important function of offering

sacrificial food to the spirits of the forefathers.

Q 24.D

o According to Transparency International, Corruption is the abuse of entrusted power for private gain. It

can be classified as grand, petty and political, depending on the amounts of money lost and the sector

where it occurs. The government of India has taken a series of measure for reducing corruption. Some of

the prominent ones include -

Indian parliament enacted the Lokpal and Lokayuktas Act in 2013. However, the act is yet to be

operationalized as the institution of Lokpal is yet to be constituted. As per the recent report, the

government has constituted eight-member search committee headed by former Supreme Court judge

Justice Ranjana Prakash Desai to recommend names for Chairperson and members. The selection

process of Lokpal is underway as per guidelines of laid down in Lokpal and Lokayuktas Act,

2013. Hence, statement 1 is correct.

India has ratified the UN Convention against Corruption in 2011. The convention binds the member-

countries to render mutual legal assistance towards prosecution of offenders as well in tracing,

freezing, and confiscating the proceeds of corruption. Hence, statement 2 is correct.

India has adopted Integrity Pact in major government Procurement activities. The goal of the Integrity

Pact is to reduce any (and almost ensure no) chances of corrupt practices during procurement through

a binding agreement between the agency and bidders for specific contracts. Central Vigilance

Commission is the nodal agency for adoption of Integrity pact in India. Hence, statement 3 is

correct.

Project Insight is an initiative by the Income-Tax Department to mine big data from social media to

scrutinize potential tax evaders. It will collate all the information available with I-T Dept. from

various sources and systematically profile individuals using PAN details. It will help in reducing

generation and use of Black-money, thus helping in reducing corruption. Hence, statement 4 is

correct.

Q 25.C

o Ministry of Electronics and Information Technology (MeitY) along with National Informatics Centre

(NIC), the nodal information technology arm of Government of India, have developed Electronic

Transaction Aggregation & Analysis Layer (eTaal) portal.

o It provides an aggregated view of e-Transactions performed through e-Governance applications

implemented including the national-level mission mode projects (MMPs) under the National e-

Governance Plan (NeGP).

o eTaal automatically pulls the e-transaction count, but not the personal details, from the applications using

web service technology.

https://telegram.me/UPSCMaterials https://telegram.me/FreeUPSCMaterials https://telegram.me/MaterialforExam

Page 12: VISION IAS - WordPress.com · 2018-11-05 · 3 ©Vision IAS Service Delivery Preparedness and achievement of Results Sound Public Grievance Redress Mechanism o Sevottam has also been

12 www.visionias.in ©Vision IAS

Q 26.B

o New telecom licenses in India are granted by Department of Telecom, Ministry of

Communications. TRAI only has a recommendatory role to advise DoT for granting the new

licenses. Hence statement 1 is not correct.

o The Telecom Regulatory Authority of India (TRAI) was formed in Jan 1997 under the Telecom

Regulatory Authority of India Act, 1997 with a view to the an effective regulatory framework and

adequate safeguards to ensure fair competition and protection of consumer interests.

o Telecom Regulatory Authority of India (TRAI) is empowered to fix tariff for telecommunication

services under Section 11(2) of TRAI Act, 1997. Tariff regulation for telecommunication services in India

was initiated by TRAI with the notification of Telecom Tariff Order, 1999. Hence statement 2 is correct.

o The Telecom Regulatory Authority of India Act was amended in the year 2000.

o The functions of the original TRAI have now been split between two bodies :

Telecom Regulatory Authority of India (TRAI): TRAI has been assigned the recommendatory and

regulatory functions

Telecom Disputes Settlement and Appellant Tribunal (TDSAT): Dispute settlement functions in the

telecom sector are with TDSAT. Hence statement 3 is not correct.

Q 27.C

o The Asia-Pacific Trade Agreement (APTA), previously named the Bangkok Agreement, was signed in

1975 as an initiative of Economic and Social Commission for Asia and the Pacific.

o Being the oldest preferential trade agreement among developing countries in Asia-Pacific, APTA

aims to promote economic development through the adoption of mutually beneficial trade liberalization

measures that will contribute to intra-regional trade expansion and provides for economic integration

through coverage of merchandise goods, services, investment and trade facilitation.

o Open to all developing member countries, APTA is a truly region-wide trade agreement spanning

East and South Asia, with the potential to expand to other sub-regions, including Central Asia and the

Pacific.

o APTA is the first plurilateral agreement among the developing countries in the region to adopt

common operational procedures for certification and verification of the origin of goods and it has the

longest effective implementation period amongst the trade agreements in the entire Asia-Pacific.

o APTA is the only operational trade agreement linking China and India, two of the fastest growing

markets in the world, and other major markets such as the Republic of Korea.

o Currently, the following six Participating States are parties to APTA:

People‟s Republic of Bangladesh

People‟s Republic of China

Republic of India

Lao People‘s Democratic Republic

Republic of Korea

Democratic Socialist Republic of Sri Lanka

o The fourth round of trade negotiations under the agreement was conducted on January 13, 2017. The

results of the fourth round of negotiations under APTA have been implemented with effect from July 1,

wherein the Indian govt. cut duties on the import of 3,142 items from the signatories to the accord,

including China.

Q 28.C

o Supreme Court in Prakash Singh versus Union of India, 2006 had laid down seven directives

on police reforms for the central and state governments.

o The 7 directives are:

o Constitute a State Security Commission (SSC) to:

Ensure that the state government does not exercise unwarranted influence or pressure on the police

Lay down broad policy guideline and

Evaluate the performance of the state police

https://telegram.me/UPSCMaterials https://telegram.me/FreeUPSCMaterials https://telegram.me/MaterialforExam

Page 13: VISION IAS - WordPress.com · 2018-11-05 · 3 ©Vision IAS Service Delivery Preparedness and achievement of Results Sound Public Grievance Redress Mechanism o Sevottam has also been

13 www.visionias.in ©Vision IAS

o Ensure that the DGP is appointed through merit based transparent process and secure a minimum tenure

of two years

o Ensure that other police officers on operational duties (including Superintendents of Police in-charge of a

district and Station House Officers in-charge of a police station) are also provided a minimum tenure of

two years

o Separate the investigation and law and order functions of the police

o Set up a Police Establishment Board (PEB) to decide transfers, postings, promotions and other service

related matters of police officers of and below the rank of Deputy Superintendent of Police and make

recommendations on postings and transfers above the rank of Deputy Superintendent of Police

o Set up a Police Complaints Authority (PCA) at state level to inquire into public complaints against police

officers of and above the rank of Deputy Superintendent of Police in cases of serious misconduct,

including custodial death, grievous hurt, or rape in police custody and at district levels to inquire into

public complaints against the police personnel below the rank of Deputy Superintendent of Police in cases

of serious misconduct

o Set up a National Security Commission (NSC) at the union level to prepare a panel for selection and

placement of Chiefs of the Central Police Organisations (CPO) with a minimum tenure of two years.

Q 29.B

o Statement 2 is not correct and statement 1 is correct: National Health Stack (NHS) is an initiative of

NITI Aayog. The National Health Stack will facilitate the collection of comprehensive healthcare data

across the country. Designed to leverage India Stack, subsequent data analysis on NHS will not only

allow policymakers to experiment with policies, detect fraud in health insurance, measure outcomes and

move towards smart policymaking, it will also engage market players (NGOs, researchers, watchdog

organizations) to innovate and build relevant services on top of the platform and fill the gaps.

o The key components of the National Health Stack are:

National Health Electronic Registries: to create a single source of truth for and manage master health

data of the nation;

A Coverage and Claims platform: building blocks to support large health protection schemes enable

horizontal and vertical expansion of RSSM by states and robust fraud detection;

A Federated Personal Health Records (PHR) Framework: to solve twin challenges of access to their

own health data by patients and availability of health data for medical research, critical for advancing

our understanding of human health;

A National Health Analytics Platform: to bring a holistic view combining information on multiple

health initiatives and feed into smart policymaking, for instance, through improved predictive

analytics;

Other horizontal Components: including, and not restricted to, unique Digital Health ID, Health Data

Dictionaries and Supply Chain Management for Drugs, payment gateways etc shared across all health

programs.

o Statement 3 is correct: The large data aggregated through this platform would be analyzed by deploying

a powerful technology arsenal from Big Data Analytics and Machine Learning all the way to

artificial intelligence and a state-of-the-art policy MarkUp Language.

Q 30.D

o National Health Policy, 2017 (NHP, 2017) launched by the Ministry of Health and Family welfare aims at

achieving universal health coverage and delivering quality health care services to all at an affordable cost.

o Some of the objectives of the policy are as follows:

The main objective of the National Health Policy 2017 is to achieve the highest possible level of

good health and well-being, through a preventive and promotive health care orientation in all

developmental policies, and to achieve universal access to good quality healthcare services

https://telegram.me/UPSCMaterials https://telegram.me/FreeUPSCMaterials https://telegram.me/MaterialforExam

Page 14: VISION IAS - WordPress.com · 2018-11-05 · 3 ©Vision IAS Service Delivery Preparedness and achievement of Results Sound Public Grievance Redress Mechanism o Sevottam has also been

14 www.visionias.in ©Vision IAS

without anyone having to face financial hardship as a consequence. Hence, statement 1 is

correct.

To provide access and financial protection at secondary and tertiary care levels, the policy

proposes free drugs, free diagnostics and free emergency care services in all public hospitals.

Hence, statement 2 is correct.

Increase Life Expectancy at birth from 67.5 to 70 by 2025.

Reduce Under Five Mortality to 23 by 2025 and MMR from current levels to 100 by 2020.

Reduce infant mortality rate to 28 by 2019.

Increase health expenditure by Government as a percentage of GDP from the existing 1.15% to 2.5 %

by 2025.

Reduce neo-natal mortality to 16 and stillbirth rate to ―single digit‖ by 2025.

Achieve global target of 2020 which is also termed as the target of 90:90:90, for HIV/AIDS

To reduce premature mortality from cardiovascular diseases, cancer, diabetes or chronic respiratory

diseases by 25% by 2025.

Relative reduction in the prevalence of current tobacco use by 15% by 2020 and 30% by 2025.

Reduction of 40% in the prevalence of stunting of under-five children by 2025.

Access to safe water and sanitation to all by 2020 (Swachh Bharat Mission).

Providing an insurance cover of five lakh to all below poverty line families is not an aim of this

policy. Hence, statement 3 is not correct.

Q 31.A

o The National Legal Services Authority (NALSA) is a statutory body constituted under the Legal Services

Authorities Act, 1987. Hence statement 1 is correct.

o It seeks to provide free Legal Services to the weaker sections of the society and to organize Lok Adalats

for amicable settlement of disputes. Hence statement 3 is correct.

o The Chief Justice of India is the Patron-in-Chief, not the Executive Chairman. Supreme Court

Judge, other than CJI, is the Executive Chairman of the Authority. Hence statement 2 is correct.

o Article 39A of the Constitution of India provides that State shall secure that the operation of the legal

system promotes justice on a basis of equal opportunity, and shall in particular, provide free legal aid, by

suitable legislation or schemes or in any other way, to ensure that opportunities for securing justice are not

denied to any citizen by reason of economic or other disability.

o Articles 14 and 22(1) also make it obligatory for the State to ensure equality before law and a legal system

which promotes justice on a basis of equal opportunity to all. Legal aid strives to ensure that this

constitutional pledge is fulfilled in its letter and spirit and equal justice is made available to the poor,

downtrodden and weaker sections of the society.

o In every State, State Legal Services Authority has been constituted to give effect to the policies and

directions of the NALSA and to give free legal services to the people and conduct Lok Adalats in the

State. The State Legal Services Authority is headed by Hon‘ble the Chief Justice of the respective High

Court who is the Patron-in-Chief of the State Legal Services Authority. In every District, District Legal

Services Authority has been constituted to implement Legal Services Programmes in the District. The

District Legal Services Authority is situated in the District Courts Complex in every District and chaired

by the District Judge of the respective district.

Q 32.A

o Pair 1 is correctly matched: The Yogyakarta Principles is a document about human rights in the areas of

sexual orientation and gender identity, published as the outcome of an international meeting of human

rights groups in Yogyakarta, Indonesia, in November 2006.

o Pair 2 is not correctly matched: The Declaration of Alma-Ata was adopted at the International

Conference on Primary Health Care (PHC) in 1978. It expressed the need for urgent action by all

https://telegram.me/UPSCMaterials https://telegram.me/FreeUPSCMaterials https://telegram.me/MaterialforExam

Page 15: VISION IAS - WordPress.com · 2018-11-05 · 3 ©Vision IAS Service Delivery Preparedness and achievement of Results Sound Public Grievance Redress Mechanism o Sevottam has also been

15 www.visionias.in ©Vision IAS

governments, all health and development workers, and the world community to protect and promote the

health of all people.

o Pair 3 is not correctly matched: The Beijing Declaration and Platform for Action is a global

commitment to achieving equality, development, and peace for women worldwide. It was adopted in

September 1995 at the Fourth World Conference on Women.

Q 33.D

o In July, a bill was introduced by the Government in Lok Sabha to establish a National Sports University

in Manipur (Imphal).

o This will be a specialized University, first of its kind, to promote sports education in the areas of sports

sciences, sports technology, sports management, and sports coaching. Besides, the proposed university

would also function as the national training center for select sports disciplines by adopting best

international practices and for matters connected therewith or incidental thereto.

Q 34.B

o The National Police Mission (NPM) was announced in 2005 by the then Prime Minister to transform the

Police Forces in the country into effective instruments for maintenance of internal security by equipping

them with the necessary material, intellectual and organizational resources and to create a "New Vision"

for the Indian police.

o The objective of this Mission is to prepare the police forces for emerging challenges, to bring about

specialization in areas like counter terrorism, counter insurgency, cyber and economic crimes,

strengthen the metropolitan and rural policing, to bring in attitudinal changes in police by transforming the

'force psychology' into 'service psychology, harness technology in aid of policing and to adopt community

policing and ensure effective delivery of services to the citizens. Hence statement 2 is correct.

o Eight Micro Missions have been created to develop projects on various subjects of Policing and Internal

Security. These projects deal with Human Resource Development, Community Policing, Communication

and Technology, Infrastructure, New Processes (Process Engineering), Proactive Policing and Visualizing

Future Challenges, and Gender Crimes and Gender-Related Issues, and Correctional Administration.

o The Mission continued to work under the aegis of Ministry of Home Affairs till December, 2008 and

was subsequently anchored in the Bureau of Police Research and Development (BPR&D), in order

to coordinate the activities of the Micro Missions and monitor the implementation of various projects in

States/UTs. Hence statement 1 is not correct.

Q 35.D

o In People‟s Union for Civil Liberties vs. Union Of India case, the Supreme Court held that a voter

could exercise the option of negative voting and reject all candidates as unworthy of being elected.

The voter could press the „None of the Above‟ (NOTA) button in the electronic voting machine

(EVM). Hence option 1 is correct.

o Section 8(4) of RPA states that if a sitting member of Parliament or state legislature is convicted

and sentenced to not less than 2 years of imprisonment shall be disqualified from being a member of

the house. However, if the member goes on appeal against his conviction within 3 months, then he shall

not be subject to disqualification. The Supreme Court in Lily Thomas vs. Union of India 2013 held

Section 8(4) as unconstitutional and void. Hence, now if a sitting member of Parliament or state

legislature is convicted and sentenced to not less than 2 years of imprisonment, he will get immediately

disqualified from being a member of the house. Hence option 2 is correct.

o In the Krishna Kumar Singh versus State of Bihar 2017, the Supreme Court observed that ordinances

are subject to judicial review, and do not automatically create enduring effects. It also places a timely

check on a power rampantly abused by governments. Hence option 3 is correct.

https://telegram.me/UPSCMaterials https://telegram.me/FreeUPSCMaterials https://telegram.me/MaterialforExam

Page 16: VISION IAS - WordPress.com · 2018-11-05 · 3 ©Vision IAS Service Delivery Preparedness and achievement of Results Sound Public Grievance Redress Mechanism o Sevottam has also been

16 www.visionias.in ©Vision IAS

Q 36.C

o Statement 1 is not correct: Government enacted POCSO Act, 2012 to protect children from offences of

sexual assault, sexual harassment and pornography, while safeguarding the interest of the child at every

stage of the judicial process, by incorporating child friendly mechanisms for reporting, recording of

evidence, investigation and speedy trial of offences through designated Special Courts. Any human being

up to the age of 18 years is recognised as a child under the POCSO Act.

o Statement 2 is not correct: An online complaint box for reporting child sexual abuse, the POCSO e-

Box was launched by the Ministry for Women and Child Development. Protection of Children from

Sexual Offences (POCSO), e-Box, is an online complaint management system for easy and direct

reporting of sexual offences against children and timely action against the offenders under the POCSO

Act, 2012. However, SHe-Box has been launched for online complaints of sexual harassment at the

workplace for all women in the country irrespective of their work status or organization.

o Statement 3 is correct: National Commission for Protection of Child Rights (NCPCR) is mandated to

monitor the implementation of the provisions of the Act.

Q 37.D

o Office of Lokpal under Lokpal and Lokayuktas Act 2013 has jurisdiction over

any person who is or has been a Prime Minister,

any person who is or has been a Minister of the Union,

any person who is or has been a member of either House of Parliament,

any Group 'A' or Group 'B' officer or equivalent or above,

any Group 'C' or Group 'D' official or equivalent,

any person who is or has been a chairperson or member or officer or employee in any body or Board or

corporation or authority or company or society or trust or autonomous body (by whatever name called)

established by an Act of Parliament or wholly or partly financed by the Central Government or

controlled by it,

any person who is or has been a director, manager, secretary or other officer of every other society or

association of persons or trust (whether registered under any law for the time being in force or not), by

whatever name called, wholly or partly financed by the Government and the annual income of which

exceeds such amount as the Central Government may, by notification, specify;

any person who is or has been a director, manager, secretary or other officer of every other society or

association of persons or trust (whether registered under any law for the time being in force or not) in

receipt of any donation from any foreign source under the Foreign Contribution (Regulation) Act, 2010

in excess of ten lakh rupees in a year or such higher amount as the Central Government may, by

notification, specify.

Q 38.D

o Statement 1 is correct: Systematic Voters‘ Education and Electoral Participation (SVEEP) is a

programme of multi interventions through different modes and media designed to educate citizens,

electors and voters about the electoral process in order to increase their awareness and participation in the

electoral processes.

o Statement 2 is correct : SVEEP is designed according to the socio-economic, cultural and demographic

profile of the state as well as history of electoral participation in previous rounds of elections and learning

thereof.

o Statement 3 is correct: Systematic Voter‘s Education and Electoral Participation (SVEEP) was launched

by Election Commission of India.

Q 39.B

o Citizens Charter is a document which represents a systematic effort to focus on the commitment of the

Organisation towards its Citizens in respects of Standard of Services, Information, Choice and

https://telegram.me/UPSCMaterials https://telegram.me/FreeUPSCMaterials https://telegram.me/MaterialforExam

Page 17: VISION IAS - WordPress.com · 2018-11-05 · 3 ©Vision IAS Service Delivery Preparedness and achievement of Results Sound Public Grievance Redress Mechanism o Sevottam has also been

17 www.visionias.in ©Vision IAS

Consultation, Non-discrimination and Accessibility, Grievance Redress, Courtesy and Value for

Money. This also includes expectations of the Organisation from the Citizen for fulfilling the

commitment of the Organisation.

o The Citizens Charter is not legally enforceable and, therefore, is non-justiciable. However, it is a tool

for facilitating the delivery of services to citizens with specified standards, quality and time frame etc.

with commitments from the Organisation and its clients. Hence statement 1 is not correct.

o „Citizen‟ with reference to Citizens Charter- The term ‗Citizen‘ in the Citizen‘s Charter implies the

clients or customers whose interests and values are addressed by the Citizen‘s Charter and,

therefore, includes not only the citizens but also all the stakeholders, i.e., citizens, customers, clients,

users, beneficiaries, other Ministries/ Departments/ Organisations, State Governments, UT

Administrations etc.

o Role of Department of Administrative Reforms and Public Grievances in Citizens Charter Initiative

in the Government Department of Administrative Reforms and Public Grievances in Ministry of

Personnel, Public Grievances and Pensions, Government of India, coordinates the efforts to formulate and

operationalize Citizen‘s Charters in Central Government, State Governments and UT Administrations. It

provides guidelines for formulation and implementation of the Charters as well as their evaluation. Hence

statement 2 is correct.

o Hence option (b) is the correct answer.

Q 40.D

o Fugitive Economic Offenders‘ Act has the following provisions:

The Act allows for a person to be declared as a fugitive economic offender (FEO) if:

an arrest warrant has been issued against him for any specified offenses and

the value involved is over Rs 100 crore and

he has left the country and refuses to return to face prosecution.

To declare a person an FEO, an application will be filed in a Special Court (designated under the

Prevention of Money-Laundering Act, 2002). The Special Court will require the person to appear at a

specified place at least six weeks from issue of notice. Proceedings will be terminated if the person

appears else he will be declared FEO by the court.

Upon declaration as an FEO, properties of a person may be confiscated and vested in the central

government, free of rights and claims in the property. Further, the FEO or any company associated

with him may be barred from filing or defending civil claims.

Appeals against the orders of the Special Court will lie before the High Court

The Act does not require the authorities to obtain a search warrant or ensure the presence of witnesses

before a search.

Q 41.C

Rashtriya Gram Swaraj Abhiyan (RGSA) is launched by Ministry of Panchayati Raj for a period of 4

years (2018-2022). It is a Centrally Sponsored Scheme (CSS). Hence, statement 1 and 2 are not correct.

This scheme extends to all States and UTs of the Country and will also include institutions of rural local

government in non-Part IX areas, where Panchayats do not exist. RGSA will help more than 2.55 lakh

Panchayati Raj Institutions (PRIs) to develop governance capabilities to deliver on SDGs through

inclusive local governance with a focus on optimum utilization of available resources. The scheme will

strengthen Gram Sabhas to function as effective institutions with social inclusion of citizens particularly

the vulnerable groups.

The scheme has both Central Component - National Level activities including "National Plan of Technical

Assistance", "Mission Mode project on e-Panchayat", "Incentivization of Panchayats" and State

component - Capacity Building of Panchayati Raj Institutions (PRIs). The Central Component will be

https://telegram.me/UPSCMaterials https://telegram.me/FreeUPSCMaterials https://telegram.me/MaterialforExam

Page 18: VISION IAS - WordPress.com · 2018-11-05 · 3 ©Vision IAS Service Delivery Preparedness and achievement of Results Sound Public Grievance Redress Mechanism o Sevottam has also been

18 www.visionias.in ©Vision IAS

fully funded by the Government of India. However, Centre: State funding pattern for State Component

will be 60:40 for all States, except North East and the Hill States where Centre: State funding pattern will

be 90:10. Hence, statement 3 is correct.

Q 42.B

Statement 1 is not correct: Child‘s Right to be adopted is not a Constitutional right, it is legal right under

the Juvenile Justice Act, 2015.

Statement 2 is correct: Central Adoption Resource Authority (CARA) is a statutory body of the

Ministry of Women & Child Development, Government of India. It functions as the nodal body for the

adoption of Indian children and is mandated to monitor and regulate in-country and inter-country

adoptions. CARA is designated as the Central Authority to deal with inter-country adoptions in

accordance with the provisions of the Hague Convention on Inter-country Adoption, 1993, ratified by

Government of India in 2003.CARA primarily deals with the adoption of an orphan, abandoned and

surrendered children through its associated / recognized adoption agencies.

Q 43.B

o Statement 1 is correct: Manual Scavengers, as defined under the ―Prohibition of Employment as Manual

Scavengers and their Rehabilitation Act, 2013‖ and their dependents, irrespective of their income, are

eligible for assistance under the Scheme. Self-Employment Scheme for Rehabilitation of Manual

Scavengers (SRMS) provide rehabilitation benefits for both identified manual scavenger as well as their

dependents. The benefits include

One time cash assistance of Rs 40,000/-

Loan upto a maximum cost of Rs. 10 lacs will be admissible under the scheme and Rs. 15 lacs in case

of sanitation related projects like Vaccum Loader, Suction Machine with Vehicle, Garbage Disposal

Vehicle, Pay & Use Toilets etc. which are extremely relevant for the target group, with high success

rate and income. Hence statement 2 is not correct.

Credit linked back end capital subsidy up to Rs 3,25,000/-

Skill development training up to two years with stipend of Rs 3,000/- per month.

o The objective of the scheme is to assist the manual scavengers, identified during various surveys, for

their rehabilitation in alternative occupations. Prohibition of Employment as Manual Scavengers and their

Rehabilitation Act, 2013 (MS Act, 2013) mandates the survey to be conducted in rural as well as urban

areas by CEO of Panchayat and CEO of Municipality respectively. The Act also makes provision that any

person working as manual scavenger can make a request to the respective CEOs for identification as

manual scavenger either during the survey or any time thereafter. The CEO shall inquire into the request

within 15 days from the date of such request.

o Statement 3 is correct: The scheme is a Central Sector Scheme and is being implemented by Ministry of

Social Justice and Empowerment.

o In view of growing incidents of deaths of manual scavengers, National Commission for Safai

Karamcharis (NCSK) organised an Awareness-cum-Health Camp for Safai Karamcharis.

Q 44.D

o The Mahatma Gandhi National Rural Employment Guarantee Act (MGNREGA) was the first law

to mandate social audit as a statutory requirement. However, even within the MGNREGA, social

audits made painfully slow progress. They faced their most trenchant opposition in Rajasthan, where the

concept was born. Elected representatives and officials reacted with intimidation, violence, and pressure

on the political leadership to stall and neutralize the process. The notable exception was undivided Andhra

Pradesh which institutionalized social audits and drew significant positive outcomes. There have been

innovative efforts in States like Sikkim, Tamil Nadu, and Jharkhand. Nationally, institutionalized social

audits have begun to make real progress only recently, with the interest and support of the office of the

Comptroller and Auditor General (CAG), and the orders of the Supreme Court. In what was a social audit

https://telegram.me/UPSCMaterials https://telegram.me/FreeUPSCMaterials https://telegram.me/MaterialforExam

Page 19: VISION IAS - WordPress.com · 2018-11-05 · 3 ©Vision IAS Service Delivery Preparedness and achievement of Results Sound Public Grievance Redress Mechanism o Sevottam has also been

19 www.visionias.in ©Vision IAS

breakthrough in 2017, Meghalaya became the first State to pass and roll out a social audit law to

cover all departments. Hence, statements 1 and 3 are correct.

o Social Audit Mechanism under MGNREGS:

State Governments are required to establish an independent organization, Social Audit Unit (SAU),

under the Mahatma Gandhi National Rural Employment Guarantee Act (MGNREGA) to facilitate the

conduct of Social Audit by Gram Sabhas.

As per Ministry of Rural Development order (April 2013), one percent of total annual

expenditure under MGNREGA in the State/UTs has to be used for meeting the cost of

establishment of a Social Audit Unit (SAU) and conducting Social Audit of MGNREGS

works. Hence statement 2 is correct.

o The Office of the CAG developed social audit rules for the MGNREGA in 2011, conducted a

performance audit in 2015, and finally, a year later formulated social audit standards in consultation with

the Ministry of Rural Development — the first time in the world. If these are followed, it can be ensured

that the social audit process is viable, credible and true to first principles of social accountability.

Q 45.D

o Statement 1 is correct: The All India Council for Technical Education (AICTE) was set up in 1945 as an

advisory body and later on in 1987 given the statutory status by an Act of Parliament (AICTE Act, 1987).

o Statement 2 is correct: The AICTE grants approval for starting new technical institutions, for the

introduction of new courses and for variation in intake capacity in technical institutions. It lays down

norms and standards for such institutions. It also ensures the quality development of technical

education through accreditation of technical institutions or programmes. In addition to its regulatory

role, the AICTE also has a promotional role which it implements through schemes for promoting technical

education for women, handicapped and weaker section of the society promoting innovations, faculty,

research, and development, giving grants to technical institutions.

o The technical institutions under the AICTE include post-graduate, undergraduate and diploma in the

whole spectrum of technical education covering engineering/technology, pharmacy, architecture, hotel

management and catering technology, management studies, computer applications and applied arts and

crafts.

o Statement 3 is correct: In order to make development a comprehensive mass movement and innovate on

all fronts, MHRD, AICTE, i4c, and Persistent Systems have come together to organize Smart India

Hackathon (SIH) 2019 - a unique Open Innovation Model for identifying new and disruptive technology

innovations to solve the challenges faced in our country. It's a non-stop product development competition,

where problem statements are posed to technology students for innovative solutions. It can help to:

Harness creativity & expertise of students

Spark institute-level hackathons

Build funnel for 'Startup India' campaign

Crowdsource solutions for improving governance and quality of life

Provide an opportunity for citizens to provide innovative solutions to India's daunting problems

o The first two editions SIH 2017 and SIH 2018 proved to be extremely successful in promoting innovation,

out-of-the-box thinking in young minds, especially engineering students from across India.

Q 46.D

o The Second Administrative Reforms Commission in its report entitled ―Citizen Centric Administration:

The Heart of Governance‖ (2009) identified five barriers to good governance in India and explained them

in the following way:

Attitudinal Problems of the Civil Servants: There is a growing concern that the Civil Services and

administration in general, have become wooden, inflexible, self-perpetuating and inward looking.

Consequently, their attitude is one of indifference and insensitivity to the needs of citizens.

https://telegram.me/UPSCMaterials https://telegram.me/FreeUPSCMaterials https://telegram.me/MaterialforExam

Page 20: VISION IAS - WordPress.com · 2018-11-05 · 3 ©Vision IAS Service Delivery Preparedness and achievement of Results Sound Public Grievance Redress Mechanism o Sevottam has also been

20 www.visionias.in ©Vision IAS

Lack of Accountability: Seldom are disciplinary proceedings initiated against delinquent

government servants and imposition of penalties is even rarer. This is primarily because, at most

levels, authority is divorced from accountability leading to a system of realistic and plausible alibis.

Red-Tapism: Bureaucracies the world over are expected to adhere to rules and procedures which

are, of course, important for good governance. However, at times, these rules and procedures are ab-

initio ill-conceived and cumbersome and, therefore, do not serve their purpose.

Low Levels of Awareness of the Rights and Duties of Citizens: Inadequate awareness about their

rights prevents citizens from holding erring government servants to account. Similarly, low levels of

compliance of rules by the citizens also act as an impediment to good governance; when citizens do

not adhere to their duties they infringe on the freedom and rights of other citizens.

Ineffective Implementation of Laws and Rules: Effective implementation of laws creates an

environment which would improve the welfare of all citizens and at the same time, encourage each

citizen to contribute his best towards the development of society. On the other hand, weak

implementation can cause a great deal of hardship to citizens and even erode the faith of the citizenry

in the government machinery. Hence all the statements are correct.

Q 47.D

o According to the Prohibition of Child Marriage Act, 2006, a child is a male who has not completed twenty

one years of age and a female who has not completed eighteen years of age.

o Statement 1 is not correct: A complaint may be made by any person having personal knowledge or

reason to believe, and a non-governmental organisation having reasonable information, relating to the

likelihood of taking place of solemnisation of a child marriage or child marriages. The Court ofthe

Judicial Magistrate of the first class or the Metropolitan Magistrate may also take suo motu cognizance on

the basis of any reliable report or information

o Statement 2 is correct: The Act specifies the appointment of Child Marriage Prohibition Officer for

whole or a part of a State by the State government.

o Statement 3 is not correct: Section 3 of the Act does not automatically makes the child marriage void

but makes it voidable at the option of the contracting party who was a child at the time of marriage.

Recently, Ministry of Women and Child Development has sought an amendment to this Section of the Act

to make child marriages ―void ab initio‖ (invalid from the outset).

Q 48.B

o Statement 1 is correct: It is an electoral system that seeks to create a representative body that reflects the

overall distribution of public support for each party. Constitution has adopted the system of proportional

representation in the case of Rajya Sabha.

o Statement 2 is not correct: In proportional representation, elections are held in a manner that ensures that

the number of seats won by a party or group of candidates is proportionate to the number of votes

received. This ensures a wider representation of diverse opinions in an elected assembly, reflecting a true

democracy. The proportional representation system rarely results in one party holding an absolute

majority. It requires political parties to compromise and build consensus. Whereas, the election of a

candidate on the basis of the majority of votes he would get is a feature of territorial representation

system.

Q 49.D

o Corporate governance broadly refers to the mechanisms, relations, and processes by which a

corporation is controlled and is directed; involves balancing the many interests of the stakeholders of a

corporation. Various committees have been constituted from time to time to deal with the issue of

corporate governance, such as:

CII Code Committee (1998)

Kumar Mangalam Birla Committee (1999)

Naresh Chandra Committee (2002)

https://telegram.me/UPSCMaterials https://telegram.me/FreeUPSCMaterials https://telegram.me/MaterialforExam

Page 21: VISION IAS - WordPress.com · 2018-11-05 · 3 ©Vision IAS Service Delivery Preparedness and achievement of Results Sound Public Grievance Redress Mechanism o Sevottam has also been

21 www.visionias.in ©Vision IAS

Narayana Murthy Committee (2003)

J.J. Irani Committee (2005)

Uday Kotak Committee (2017)

Q 50.A

o Recently the government announced a comprehensive plan Project Sashakt for the resolution of

stressed assets in the banking sector.

o It aims to strengthen the credit capacity, credit culture and credit portfolio of public sector banks.

o It is a five-pronged strategy towards resolution of stressed assets, as recommended by Sunil Mehta

Committee.

Small and Medium Enterprise (SME) resolution approach

Bank-led resolution approach

National Company Law Tribunal /Insolvency and Bankruptcy Code approach

Asset-trading platform

Q 51.B

o Statement 1 is correct: The Conduct Rules, contained in the Central Services (Conduct) Rules, 1964 and

analogous rules applicable to members of the All India Services or employees of various State

Governments. The code of behaviour as enunciated in the Conduct Rules, while containing some general

norms like 'maintaining integrity and absolute devotion to duty' and not indulging in 'conduct unbecoming

of a government servant' is generally directed towards cataloguing specific activities deemed undesirable

for government servants. These conduct rules do not constitute a code of ethics.

o Statement 2 is not correct: There is no Code of Ethics prescribed for civil servants in India although

such Codes exist in other countries.

o Statement 3 is correct: The Central and State Vigilance Commission are examples of other oversight

mechanisms that are autonomous but lie within the framework of the State. Analysts have categorized

these accountability mechanisms into ―horizontal‖ accountability mechanisms which refer to those located

within the State as against 'vertical' accountability mechanisms which are those outside the State and

include the media, civil society and citizen.

Q 52.C

o The state executive consists of the Governor, the Chief Minister, the Council of Ministers and the

Advocate General of the State.

o Statements 1 and 2 are correct: In the performance of his official duties, the advocate general is

entitled to appear before any court of law within the state. Further, he has the right to speak and to

take part in the proceedings of both the Houses of the state legislature or any committee of the state

legislature of which he may be named a member, but without a right to vote. He enjoys all the privileges

and immunities that are available to a member of the state legislature.

Q 53.C

o Central Drugs Standard Control Organization is a drug regulatory agency under Central Government

responsible for approval of licenses of specified categories of Drugs. Under the Drug and Cosmetics Act,

1940.

o The central authorities are responsible for approval of new drugs, clinical trials in the country,

laying down the standards for drugs, control over the quality of imported drugs, coordination of the

activities of State Drug Control Organizations and providing expert advice with a view of bringing

about the uniformity in the enforcement of the Drugs and Cosmetics Act. Hence, statement 1 is

correct.

https://telegram.me/UPSCMaterials https://telegram.me/FreeUPSCMaterials https://telegram.me/MaterialforExam

Page 22: VISION IAS - WordPress.com · 2018-11-05 · 3 ©Vision IAS Service Delivery Preparedness and achievement of Results Sound Public Grievance Redress Mechanism o Sevottam has also been

22 www.visionias.in ©Vision IAS

o The state authorities on the other hand are concerned with the regulation of manufacture, sale and

distribution of Drugs.

o Central Drugs Standard Control Organization (CDSCO) is controlled and governed by Directorate

General of Health Services which comes under Ministry of Health and Family Welfare,

Government of India. Hence, statement 2 is correct.

o The headquarter of the Central Drugs Standard Control Organization is located at New Delhi, while it has

multiple zonal offices throughout India.

Q 54.D

o Governance Knowledge Centre (GKC): The Department of Administrative Reforms and Public

Grievances has taken up the initiative to design and develop a web-based repository of good

governance initiatives and best practices. The GKC encompasses web-based digital repository put

together by support team comprising domain experts, resource persons, analysts along with technical

professionals who continuously ensure dynamic updation of Knowledge Resources and Case Studies

relevant to the profile of users visiting the repository. The Digital Repository is envisaged as a tool to

enable capturing, organising and storing for easy retrieval of digital contents with respect to the various

selected case studies of ―Good Governance Practices‖ in India and abroad.

o The GKC Portal aims to assist civil servants to seek practical and implementable solutions to the

day-to-day challenges they face. It serves as a platform for collaborative knowledge exchange for the

improvement of governance. It also offers a widespread and reputed lot of governance knowledge that

civil society can utilise to understand the nuances of civil service practices and reforms. At present, the

portal consists of 240 well documented best practices and 544 case studies, working papers, toolkits,

policy briefings, etc.

o Hence all the statements are correct.

Q 55.B

o E-Governance has played a key role in embarking the journey to achieve the goal of ‗Minimum

Government, Maximum Governance‘. It has helped the Government to enhance the coverage, increase

transparency, improve response to citizens and lower costs; on the other hand has facilitated citizens to

have better access, equity & social empowerment.

o Department of Administrative Reforms & Public Grievances (DARPG), Ministry of Personnel,

Public Grievances & Pensions, and Ministry of Electronics and Information Technology (MeitY),

have been jointly organizing the National Conference on eGovernance every year in partnership

with one State Government since 1997.

o The DARPG, Government of India, along with the Ministry of Electronics & Information Technology,

Government of India in association with Government of Telangana organized the 21st National

Conference on e-Governance on February 26- 27, 2018 in Hyderabad, Telangana. The theme of the

Conference this year was, „Technology for accelerating Development‟.

Q 56.B

o Article 350B. Special Officer for linguistic minorities.-

There shall be a Special Officer for linguistic minorities to be appointed by the President.

It shall be the duty of the Special Officer to investigate all matters relating to the safeguards provided

for linguistic minorities under this Constitution and report to the President upon those matters at

such intervals as the President may direct, and the President shall cause all such reports to be laid

before each House of Parliament, and sent to the Governments of the States concerned.

Functions of commissioner for linguistic minorities in india:

o To investigate all matters related to safeguards provided to the linguisticminorities.

o Report to the President of India regarding the status of implementation of the Constitutional and the

nationally agreed Scheme of Safeguards for the linguistic minorities.

https://telegram.me/UPSCMaterials https://telegram.me/FreeUPSCMaterials https://telegram.me/MaterialforExam

Page 23: VISION IAS - WordPress.com · 2018-11-05 · 3 ©Vision IAS Service Delivery Preparedness and achievement of Results Sound Public Grievance Redress Mechanism o Sevottam has also been

23 www.visionias.in ©Vision IAS

o Monitoring status of implementation of Safeguards through Questionnaires, Visits, Conferences,

Seminars, Meetings, Review Mechanism, etc.

o Equality before law (Article 14), prohibition of discrimination on grounds of religion, race, caste, sex or

place of birth (Article 15) and equality of opportunity in matters of public employment (Article 16) also

operate as safeguards for linguistic minorities.

Q 57.C

o Recently a cluster of buildings have been declared a UNESCO World Heritage Site

in Mumbai. These buildings belong to Victorian Gothic and Art Deco style.

o The buildings of Victorian Gothic style, such as Old Secretariat (1857-74), University Library and

Convention Hall (1874-78), the Bombay High Court (1878), the Public Works Department Office (1872),

Watson‘s Hotel (1869), David Sasoon Library (1870), the Elphinstone College (1888) were constructed in

second half of 19th century. The buildings of Art Deco style, such as the buildings on the first row of

Marine Drive, the Regal and Eros Cinemas were constructed in early decades of 20th century.

o The High Victorian Gothic style developed in England around the midpoint of the 19th century. English

architect John Ruskin, author of "The Seven Lamps of Architecture" (1849) was a major proponent of the

style, finding "structural colouration" superior to superficially applied colour. Initially, this style was

inspired by English medieval architecture, but later it drew from medieval French and German building

traditions as well.

o The High Victorian Gothic style was used mostly for large scales public buildings like schools, churches,

or government offices, but was sometimes chosen for mansions or homes of substantial size. Always

executed in brick or stone, High Victorian Gothic buildings are distinguished by the use of polychrome

bands of decorative masonry. Stone quoins pressed brick, and terra cotta panels were commonly used.

Windows and doors were accented with brick or stone trim, often in contrasting colours. The Gothic

pointed arch may be present at windows, entrances, and decorative dormers and cross gables. Round

turrets with corbelled brickwork and conical roofs are common to this style as well.

Q 58.A

o Statement 1 is not correct: The expression "office of profit" has not been defined in the Constitution

or in the Representation of the People Act, 1951. They have been mentioned in the Article 102 and 191

of the Constitution.

o According to Article 102 (1) (a), a person shall be disqualified as a member of Parliament for holding

any office of profit under the government of India or the government of any state, "other than an office

declared by Parliament by law not to disqualify its holder". Article 191 (1) (a) has a similar provision for

the members of state assemblies.

o However, based on past judgments, the Election Commission has noted five below tests for what

constitutes an office of profit:

Whether the government makes the appointment

Whether government has the right to remove or dismiss the holder.

Whether the government pays remuneration.

What the functions of the holder are.

Does the government exercise any control over the performance of these functions.

o Statement 2 is not correct: Articles 102 and 191 clarify that "a person shall not be deemed to hold an

office of profit under the government of India or the government of any state by reason only that he is a

minister".

o Statement 3 is correct: Under Article 102 and Article 191 of the Constitution both parliament and state

legislatures have the power to exempt an office from the purview of the office of profit.

https://telegram.me/UPSCMaterials https://telegram.me/FreeUPSCMaterials https://telegram.me/MaterialforExam

Page 24: VISION IAS - WordPress.com · 2018-11-05 · 3 ©Vision IAS Service Delivery Preparedness and achievement of Results Sound Public Grievance Redress Mechanism o Sevottam has also been

24 www.visionias.in ©Vision IAS

Q 59.C

o In order to make the Digital India programme the most efficacious initiative, the Government of India

defines the nine pillars of Digital India.

o ―Broadband Highway,‖ the first pillar of Digital India programme, plans to connect 2.5 lakh village

panchayats via National Optical Fibre Network (NOFN). This will empower the rural population to access

government programmes easily and effectively.

o Under the second pillar, the initiative is to focus on network penetration and fill the gaps in

connectivity in the country. All together 42,300 uncovered villages will be covered for providing

universal mobile connectivity in the country.

o Under the third pillar ―Public Internet Access Programme,‖ the government plans to create 2.5 lakh

common service centres (CSCs), mapping one CSC to each Gram Panchayat.

o The fourth pillar ―e-Governance– Reforming Government through Technology‖ enables the government

to use IT to simplify and transform government processes more efficiently.

o The fifth pillar of Digital India programme is ―e-Kranti – Electronic delivery of services‖ to deliver good

governance to all citizens in India. "Information for All,‖

o The sixth pillar of the Digital India programme drives transparency in governance. Under Open Data

Platform ministries/departments release information proactively for public use.

o The seventh and the most important pillar of the Digital India programme is “Electronics

Manufacturing”. This pillar focuses on promoting electronics manufacturing in the country with

the target of NET ZERO Imports by 2020

o The eighth pillar “IT for Jobs” focuses on providing necessary training to the youth in smaller

towns and villages for availing employment opportunities in the IT/ITES sector.

o The ninth pillar ―Early Harvest Programmes‖ comprises the projects which can be implemented within a

short timeline which include IT platform for messages, wi-fi in all universities, secure email within

government, public wi-fi hotspots, and SMS-based weather information.

Hence, all the statements are correct.

Q 60.C

o Statement 1 is not correct: The National Green Tribunal was established on 18.10.2010 under the

National Green Tribunal Act 2010 for effective and expeditious disposal of cases relating to

environmental protection and conservation of forests and other natural resources including enforcement of

any legal right relating to environment and giving relief and compensation for damages to persons and

property and for matters connected therewith or incidental thereto.

o It is a specialized body equipped with the necessary expertise to handle environmental disputes involving

multi-disciplinary issues. The Tribunal shall not be bound by the procedure laid down under the Code of

Civil Procedure, 1908, but shall be guided by principles of natural justice. The Tribunal's dedicated

jurisdiction in environmental matters shall provide speedy environmental justice and help reduce the

burden of litigation in the higher courts.

o Statement 2 is correct: The Tribunal is mandated to make and endeavour for disposal of

applications or appeals finally within 6 months of filing of the same.

o Statement 3 is correct: Initially, the NGT is proposed to be set up at five places of sittings and will

follow circuit procedure for making itself more accessible. New Delhi is the Principal Place of Sitting of

the Tribunal and Bhopal, Pune, Kolkata and Chennai shall be the other four place of sitting of the

Tribunal

Q 61.B

o 3rd tier of governance in both rural and urban areas [local bodies] was given constitutional backing by the

73rd and 74th constitutional amendments. It envisaged the setting up of State Election Commission [SEC]

https://telegram.me/UPSCMaterials https://telegram.me/FreeUPSCMaterials https://telegram.me/MaterialforExam

Page 25: VISION IAS - WordPress.com · 2018-11-05 · 3 ©Vision IAS Service Delivery Preparedness and achievement of Results Sound Public Grievance Redress Mechanism o Sevottam has also been

25 www.visionias.in ©Vision IAS

consisting of a State Election Commissioner to be appointed by the Governor of the state for conducting

these elections. Hence statement 1 is correct.

o State Election Commissioner is an independent officer and is not linked to nor is this officer under the

control of the Election Commission of India. Hence statement 2 is not correct.

o The State Election Commissioner cannot be removed from the office except in the manner and on the

grounds prescribed for the removal of a judge of the state high court. And a judge of a high court can be

removed from his office by the president on the recommendation of the Parliament. This means that a

state election commissioner cannot be removed by the governor, though appointed by him. Hence

statement 3 is correct.

Q 62.B

o The Chairman and members of the National Human Rights Commission are appointed by the President on

the recommendations of a six-member committee consisting of the Prime Minister as its head,

the Speaker of the Lok Sabha, the Deputy Chairman of the Rajya Sabha, leaders of the Opposition

in both the Houses of Parliament and the Central home minister. Further, a sitting judge of the

Supreme Court or a sitting chief justice of a high court can be appointed only after consultation with the

chief justice of India.

Q 63.D

o The National Policy on Older Persons was announced by the Government of India in the year 1999.

Subsequently, a new policy known as the ―National Policy for Senior Citizens‖ was announced in 2011

based on the foundation of demographic explosion among the elderly, the changing economy and social

milieu, advancement in medical research and high levels of destitution among the elderly poor. The policy

defines senior citizens as those above 60 years and ‗Oldest Old‘ as 80 years and above.

o Statement 1 is correct: It also promotes the concept of „Ageing in Place‟ which involves ageing in

own home, housing, income security and homecare services, old age pension and access to

healthcare insurance schemes and other programmes and services to facilitate and sustain dignity

in old age. In addition, it also acknowledges the problems of elderly women are exacerbated by a lifetime

of gender-based discrimination, often stemming from deep-rooted cultural and social bias.

o Statement 2 is correct: The policy will promote measures to create avenues for continuity in

employment and/or post-retirement opportunities. Directorate of Employment is created to enable seniors

to find re-employment.

o Statement 3 is correct: National Council for Senior Citizens, headed Minister for Social Justice and

Empowerment monitors the implementation of the policy and advise the government on concerns of

senior citizens.

Q 64.C

o As a constitutional functionary, CAG is primarily entrusted with the responsibility to audit the accounts

and related activities of the three tiers of Government – Federal, Provincial and Local; the State owned

public sector commercial enterprises; and autonomous bodies financed by the Federal and Provincial

Governments. His reports are laid before the Parliament and Legislatures of the Provinces.

o [Articles 244(2) and 275(1)]

o District and Regional Funds—

There shall be constituted for each autonomous district, a District Fund and for each autonomous

region, a Regional Fund.

The Governor may make rules for the management of the District Fund, or, as the case may be, the

Regional Fund.

The accounts of the District Council or, as the case may be, the Regional Council shall be kept in such

form as the Comptroller and Auditor General of India may, with the approval of the President,

prescribe.

https://telegram.me/UPSCMaterials https://telegram.me/FreeUPSCMaterials https://telegram.me/MaterialforExam

Page 26: VISION IAS - WordPress.com · 2018-11-05 · 3 ©Vision IAS Service Delivery Preparedness and achievement of Results Sound Public Grievance Redress Mechanism o Sevottam has also been

26 www.visionias.in ©Vision IAS

The Comptroller and Auditor-General shall cause the accounts of the District and Regional

Councils to be audited in such manner as he may think fit, and the reports of the Comptroller

and Auditor-General relating to such accounts shall be submitted to the Governor who shall

cause them to be laid before the Council. Hence, statement 1 is correct.

o In addition to the legal and regulatory audit, the CAG can also conduct the propriety audit, that is, he can

look into the ‗wisdom, faithfulness and economy‘ of government expenditure and comment on the

wastefulness and extravagance of such expenditure. However, unlike the legal and regulatory audit, which

is obligatory on the part of the CAG, the propriety audit is discretionary. Hence, statement 2 is

correct.

Q 65.B

o The principal-agent problem arises when there is a conflict of interest between an entity (principal)

and the party it hires (agent) as its representative (to work in its interest). The principal-agent

problem occurs when there is an environment in which an agent's incentives don't align with those of the

principal.

o Principal-agent problem is also present in the working of a democracy. In democracies, people elect their

own representatives and these representatives are expected to take decisions that benefit their electorate.

However, after getting elected, many representatives take decisions that benefit them even if it goes

against the interests of their electorate, like increasing their salaries and allowances when the general

poverty level is high.

Q 66.D

o Electronically Transmitted Postal Ballot System (ETPBS) is an electronic voting system developed by

Election Commission of India with the help of Centre for Development of Advanced Computing (C-

DAC), for the use of service voters. Hence statement 1 is not correct.

o Under this system, the service voters will be receiving the downloadable postal ballot through two-layer

secure electronic medium i.e. secured ‗One Time Password (OTP)‘ for downloading the encrypted files

sent through mobile/e-mail and the system generated unique ―Personal Identification Number" (PIN) for

decrypting and printing each and every single postal ballot to the individual service voter/ authorized

person. The casted votes by service voters would be received by returning officer through the post (and

not electronically). Hence Statement 2 is not correct.

o This system can only be used by electors in the army, paramedical staff and other Central and state

services, posted outside their constituency. It is not available for the general public. Hence statement 3 is

correct.

Q 67.A

o Statement 1 is not correct: The chairman and members of the Commission hold office for a term of six

years or until they attain the age of 65 years, whichever is earlier. However, they can relinquish their

offices at any time by addressing their resignation to the president.

o Statement 2 is not correct: President need to consult SC only in the case of misbehavior. The President

can remove the chairman or any other member of UPSC from the office under the following

circumstances:(a) If he is adjudged an insolvent (that is, has gone bankrupt);(b) If he engages, during his

term of office, in any paid employment outside the duties of his office; or(c) If he is, in the opinion of the

president, unfit to continue in office by reason of infirmity of mind or body.In addition to these, the

president can also remove the chairman or any other member of UPSC for misbehavior. President has to

refer the matter to the Supreme Court for an inquiry. If the Supreme Court, after the inquiry, upholds the

cause of removal and advises so, the president can remove the chairman or a member. Under the

provisions of the Constitution, the advice tendered by the Supreme Court in this regard binding on the

president.

o Statement 3 is correct: It is consulted, not mandatorily, on all matters relating to methods of recruitment

to civil services and for civil posts

https://telegram.me/UPSCMaterials https://telegram.me/FreeUPSCMaterials https://telegram.me/MaterialforExam

Page 27: VISION IAS - WordPress.com · 2018-11-05 · 3 ©Vision IAS Service Delivery Preparedness and achievement of Results Sound Public Grievance Redress Mechanism o Sevottam has also been

27 www.visionias.in ©Vision IAS

Q 68.D

o Statement 1 is correct: In pursuance of an appeal by the then Prime Minister, Pt. Jawaharlal Nehru (first

Prime Minister) in January 1948, the Prime Minister's National Relief Fund (PMNRF) was established

with public contributions to assist displaced persons from Pakistan. The resources of the PMNRF are now

utilized primarily to render immediate relief to families of those killed in natural calamities like floods,

cyclones and earthquakes, etc. and to the victims of the major accidents and riots. Assistance from

PMNRF is also rendered, to partially defray the expenses for medical treatment like heart surgery, kidney

transplantation, cancer treatment and acid attack etc.

o Statement 2 is correct: PMNRF accepts only voluntary donations by individuals and institutions.

Contributions flowing out of budgetary sources of Government or from the balance sheets of the public

sector undertakings are not accepted. Conditional contributions, where the donor specifically mentions

that the amount is meant for a particular purpose, are not accepted in the Fund.

o Statement 3 is correct: The disbursement out of the fund is made at the discretion of the Prime Minister,

and in accordance with the Prime Minister's directions.

Q 69.B

o Statement 1 is not correct: The Model Code of Conduct is a set of guidelines formulated by the Election

Commission of India. The MCC for the guidance of political parties and candidates is a set of norms

which has been evolved with the consensus of political parties who have consented to abide by the

principles embodied in the said code and also binds them to respect and observe it in its letter and spirit.

o Statement 2 is not correct: MCC evolved by the Election Commission on the basis of a consensus

among political parties and are a set of broad guidelines as to how the political parties and candidates

should conduct themselves during the election campaign. They have not been given statutory backing in

the Representation of the People Act, 1951.

o The Model Code of Conduct is enforced from the date of announcement of election schedule by the

Election Commission and is operational till the process of elections are completed.

o Statement 3 is correct: Applicability of MCC

During general elections to the House of People (Lok Sabha), the code is applicable throughout the

country.

During general elections to the Legislative Assembly (Vidhan Sabha), the code is applicable in the

entire State.

During bye-elections, the code is applicable in the entire district or districts in which the

constituency falls.

Q 70.D

o People‘s participation in development administration means the direct involvement of people in the

process of administering development programmes meant for bringing about socio-economic changes in

society.

o People‘s participation in development administration is beneficial in various respects:

It provides the administration with a wealth of information on local socio-cultural, economic,

ecological, and technical conditions. This information is highly useful in the process of planning,

programming, and implementation of development programmes.

It leads to the selection of those projects which are of direct relevance to the people.

It facilitates the mobilization of local resources in the form of cash, labor, materials and so on which

are very essential for the programme‘s success. Hence statement 1 is correct.

It acts as a safeguard against the abuse of administrative authority and thus reduces the scope for

corruption in the operation of programmes.

It prevents the hijacking of programme benefits by richer and powerful sections due to the

involvement of poorer and weaker sections of the society. Thus, it leads to the equitable distribution

of benefits.

https://telegram.me/UPSCMaterials https://telegram.me/FreeUPSCMaterials https://telegram.me/MaterialforExam

Page 28: VISION IAS - WordPress.com · 2018-11-05 · 3 ©Vision IAS Service Delivery Preparedness and achievement of Results Sound Public Grievance Redress Mechanism o Sevottam has also been

28 www.visionias.in ©Vision IAS

It makes the local community easily accept the developmental change and more tolerable to mistakes

and failures.

It reduces the financial burden on government by sustaining the programmes even after the

withdrawal of its support. They can be managed by the volunteers or community-based

workers. Hence, statement 2 is correct.

It enhances the ability and competence of the people to assume responsibility and solve their own

problems. It develops a spirit of self-reliance, initiative, and leadership among the people.

It promotes esprit de corps in the community and thus strengthens democracy at the grassroots

level. Hence statement 3 is correct.

Q 71.A

o The Constitution (97th Amendment) Act, 2011 relates to the cooperative societies working in India. The

Amendment has brought about a number of significant changes in the Indian Constitution. A new Part

IXB was inserted. The word ―cooperatives‖ was added after ―unions and associations‖ in Art. 19(1)(c)

under Part III of the Constitution. This enables all the citizens to form cooperatives by giving it the

status of fundamental right of citizens. Also, a new Article 43B was added in the Directive Principles of

State Policy (Part IV) regarding the ―promotion of cooperative societies‖.

o Statement 1 is correct: Part IXB contains provisions relating to the incorporation, board structure,

election of members and board directors, application of this part among others, etc in order to bring about

uniformity in the process of election of its members and board of directors. The board of directors of a

cooperative shall consist of such number of directors as may be provided by the Legislature of a State, by

law: Provided that the maximum number of directors of a co-operative society shall not exceed twenty-

one. Provided further that the Legislature of a State shall, by law, provide for the reservation of one

seat for the Scheduled Castes or the Scheduled Tribes and two seats for women on board of every

co-operative society consisting of individuals as members and having members from such class or

category of persons.

o Statement 2 is not correct: The superintendence, direction and control of the preparation of electoral

rolls for, and the conduct of, all ejections to a co-operative society shall vest in such an authority or body,

as may be provided by the Legislature of a State, by law.

Q 72.B

o Article 324 to 329 of Part XV of the Constitution deals with the electoral system in our country.

Constitution allows Parliament to make provisions in all matters relating to elections to the Parliament and

State Legislatures. In exercise of this power, the Parliament has enacted laws like Representation of the

People Act 1950 (RPA Act 1950), Representation of the People Act 1951 (RPA Act 1951) and

Delimitation Commission Act of 1952, etc

o Some of the salient features of Representation People Act, 1951 are:-

As per Section 33 (7) of Representation People Act, 1951, a person cannot contest from more than

two constituencies for a Lok Sabha/Vidhan Sabha election. Hence statement 1 is not correct.

For contesting an election as a candidate a person must be registered as a voter. Sec 4 (d) of

Representation People Act, 1951 precludes a person from contesting unless he is an elector in any

parliamentary constituency. Section 5 (c) of R. P. Act, 1951 has a similar provision for Assembly

Constituencies. Hence statement 2 is correct.

According to section 62(5) of the Representation of the People Act, 1951, no person shall vote at

any election if he is confined in a prison, whether under a sentence of imprisonment or

transportation or otherwise, or is in the lawful custody of the police. Hence statement 3 is not

correct.

https://telegram.me/UPSCMaterials https://telegram.me/FreeUPSCMaterials https://telegram.me/MaterialforExam

Page 29: VISION IAS - WordPress.com · 2018-11-05 · 3 ©Vision IAS Service Delivery Preparedness and achievement of Results Sound Public Grievance Redress Mechanism o Sevottam has also been

29 www.visionias.in ©Vision IAS

Q 73.C

o The E-Government Development Index (EGDI) presents the state of E-Government Development of the

United Nations Member States. Along with an assessment of the website development patterns in a

country, the E-Government Development index incorporates the access characteristics, such as the

infrastructure and educational levels, to reflect how a country is using information technologies to

promote access and inclusion of its people.

o E-Government Development Index (EGDI) is biannually presented by the United Nations

Department of Economic and Social Affairs (UN DESA). Hence statement 2 is correct.

o EGDI is a composite measure of three important dimensions of e-government, namely:

o Provision of online services - Online Service Index

o Telecommunication connectivity - Telecommunication Index

o Human capacity - Human Capital Index

o Hence statement 1 is correct.

o EGDI Rankings 2018

Of the 193 member countries assessed in the United Nations E-Government Survey 2018, India was

in 96th place in the E-Government Development Index. In 2016, India was ranked 107th.

Q 74.C

o Both the statements are correct.

o Human Rights are non-discriminatory and equal: Non-discrimination is a cross-cutting principle in

international human rights law. The principle is present in all the major human rights treaties and provides

the central theme of some of international human rights conventions such as the International Convention

on the Elimination of All Forms of Racial Discrimination and the Convention on the Elimination of All

Forms of Discrimination against Women.

o The principle applies to everyone in relation to all human rights and freedoms and it prohibits

discrimination on the basis of a list of non-exhaustive categories such as sex, race, colour and so on. The

principle of non-discrimination is complemented by the principle of equality, as stated in Article 1 of the

Universal Declaration of Human Rights: ―All human beings are born free and equal in dignity and rights.‖

o Human rights entail both rights and obligations. States assume obligations and duties under

international law to respect, to protect and to fulfil human rights. The obligation to respect means that

States must refrain from interfering with or curtailing the enjoyment of human rights. The obligation to

protect requires States to protect individuals and groups against human rights abuses. The obligation to

fulfil means that States must take positive action to facilitate the enjoyment of basic human rights. At the

individual level, while we are entitled our human rights, we should also respect the human rights of

others.

o All human rights are indivisible, whether they are civil and political rights, such as the right to life,

equality before the law and freedom of expression; economic, social and cultural rights, such as the rights

to work, social security and education, or collective rights, such as the rights to development and self-

determination, are indivisible, interrelated and interdependent. The improvement of one right facilitates

advancement of the others. Likewise, the deprivation of one right adversely affects the others.

o Human rights are universal and inalienable: The principle of universality of human rights is the

cornerstone of international human rights law. This principle, as first emphasized in the Universal

Declaration on Human Rights in 1948, has been reiterated in numerous international human rights

conventions, declarations, and resolutions.

o They should not be taken away, except in specific situations and according to due process. For example,

the right to liberty may be restricted if a person is found guilty of a crime by a court of law.

Q 75.A

o Statement 1 is not correct: The Election Commission of India registers political parties for the purpose

of elections and grants them recognition as national or state parties on the basis of their poll performance.

https://telegram.me/UPSCMaterials https://telegram.me/FreeUPSCMaterials https://telegram.me/MaterialforExam

Page 30: VISION IAS - WordPress.com · 2018-11-05 · 3 ©Vision IAS Service Delivery Preparedness and achievement of Results Sound Public Grievance Redress Mechanism o Sevottam has also been

30 www.visionias.in ©Vision IAS

o Statement 2 is not correct: At the state level, the Election Commission is assisted by the chief electoral

officer who is appointed by the chief election commissioner in consultation with the state government.

o Statement 3 is correct: The Returning Officer of a parliamentary or assembly constituency is responsible

for the conduct of elections in the parliamentary or assembly constituency concerned as per section 21 of

the Representation of the People Act 1951. The Election Commission of India nominates or designates

an officer of the Government or a local authority as the Returning Officer for each of the assembly

and parliamentary constituencies in consultation with the State Government/Union Territory

Administration. In addition, the Election Commission of India also appoints one or more Assistant

Returning Officers for each of the assembly and parliamentary constituencies to assist the Returning

Officer in the performance of his functions in connection with the conduct of elections.

Q 76.C

o Narcotics Control Bureau NCB is the nodal drug law enforcement and intelligence agency of India

responsible for fighting drug trafficking and the abuse of illegal substances. It functions under Union

Ministry of Home Affairs (MHA). It is Headquartered in New Delhi.

o The Research and Analysis Wing (R&AW or RAW) is the foreign intelligence agency of India. It was

established in 1968 following the intelligence failures of the Sino-Indian war, which persuaded the

Government of India to create a specialized, independent agency dedicated to foreign intelligence

gathering; previously, both domestic and foreign intelligence had been the purview of the Intelligence

Bureau. Headquartered in New Delhi. The head of RAW is designated Secretary in the Cabinet

Secretariat and is under the direct command of the Prime Minister and reports on an

administrative basis to the Cabinet Secretary of India, who reports to the Prime Minister.

o The Indian Coast Guard protects India's maritime interests and enforces maritime law, with jurisdiction

over the territorial waters of India, including its contiguous zone and exclusive economic zone. The Indian

Coast Guard was formally established on 18 August 1978 by the Coast Guard Act, 1978 of the Parliament

of India as an independent Armed force of India. It operates under the Ministry of Defence.

o The Indian Computer Emergency Response Team( CERT-In) is an office within the Ministry of

Electronics and Information Technology. It is the nodal agency to deal with cyber security threats like

hacking and phishing. It strengthens the security-related defense of the Indian Internet domain.

Q 77.C

o Statement 1 is not correct: 'Sukanya Samriddhi Yojna‟ is a small deposit scheme for girl child,

launched as a part of the „Beti Bachao Beti Padhao' campaign on 22 January 2015. 'Sukanya

Samriddhi Account' can be opened at any time from the birth of a girl child till she attains the age of 10

years, with a minimum deposit of Rs 250.

o Although, initially the government has put the minimum deposit limit of Rs 1,000 but recently, the limit

was lowered down to Rs 250 only. Government has recently slashed the minimum deposit for the

Sukanya Samriddhi Yojana in order to increase the enrollment in the scheme.

o A maximum of Rs 1.5 lakh can be deposited during a financial year. The account can be opened in any

post office or authorised branches of commercial banks. To motivate parents to open an account in the

name of a girl child and for her welfare to deposit maximum of their savings upto the prescribed limits,

higher rates of interest is given on the deposits on annually compounded basis with income tax

concession. The account will remain operative for 21 years from the date of opening of the account or

marriage of the girl child after attaining 18 years of age.

o Statement 2 is correct: It does not allow the withdrawal before the girl child attains the age of 18

years. The provision of not allowing withdrawal from the account till the age of 18 has been kept to

prevent early marriage of girls. To meet the requirement of higher education expenses, partial withdrawal

of 50 per cent of the balance would be allowed after the girl child has attended 18 years of age.

o Statement 3 is not correct: Sukanya Samriddhi Scheme does not have such provisions for SC/ST or

minority girls.

https://telegram.me/UPSCMaterials https://telegram.me/FreeUPSCMaterials https://telegram.me/MaterialforExam

Page 31: VISION IAS - WordPress.com · 2018-11-05 · 3 ©Vision IAS Service Delivery Preparedness and achievement of Results Sound Public Grievance Redress Mechanism o Sevottam has also been

31 www.visionias.in ©Vision IAS

Q 78.C

o Bar association is a professional body of lawyers who are enrolled as it's members.

o The Bar Council of India is a statutory body created by Parliament under Advocates Act, 1961 to regulate

and represent the Indian bar. Hence statement 1 is correct.

o It does not regulate the conduct of Judges. Hence statement 2 is not correct.

o It performs the regulatory function by prescribing standards of professional conduct and etiquette and by

exercising disciplinary jurisdiction over the bar.

o It sets standards for legal education and grants recognition to Universities whose degree in law will

serve as qualification for enrolment as an advocate. Hence statement 3 is correct. It performs certain

representative functions by protecting the rights, privileges, and interests of advocates and through the

creation of funds for providing financial assistance to organize welfare schemes for them. The following

statutory functions are covered under Section 7 of Advocates Act, 1961:

To lay down standards of professional conduct and etiquette for advocates.

To lay down procedure to be followed by its disciplinary committee and the disciplinary committees

of each State Bar Council.

To safeguard the rights, privileges and interests of advocates.

To promote and support law reform.

To deal with and dispose of any matter which may be referred to it by a State Bar Council.

To promote legal education and to lay down standards of legal education. This is done in consultation

with the Universities in India imparting legal education and the State Bar Councils.

To recognize Universities whose degree in law shall be a qualification for enrolment as an advocate.

The Bar Council of India visits and inspects Universities, or directs the State Bar Councils to visit and

inspect Universities for this purpose.

To conduct seminars and talks on legal topics by eminent jurists and publish journals and papers of

legal interest.

To organize legal aid to the poor.

Q 79.A

o National e-Vidhan Application (NeVA) - NeVA, an initiative by the Ministry of Parliamentary

Affairs, aims to make all the Legislatures of the country paperless by making the proceedings of the

Houses digital. It is an attempt to provide the information about the functioning of the House to the

common citizen at the click of a button. The iOS and Google app of NeVA along with the NeVA Website

will act as a repository of data related to the business of all Legislatures in the country in a uniform

manner.

o UMANG (Unified Mobile Application for New-age Governance) is envisaged to make e-governance .

It is developed by Ministry of Electronics and Information Technology (MeitY) and National e-

Governance Division (NeGD) to drive Mobile Governance in India.UMANG provides a single platform

for all Indian Citizens to access pan India e-Gov services ranging from Central to Local Government

bodies and other citizen centric services. UMANG intends to provide major services offered by Central

and State Government departments, Local bodies and other utility services from private organizations.

o Legal Information Management & Briefing System (LIMBS) – It is an initiative under Ministry of

Law and Justice that endeavors to bring court cases, where the government is a respondent, on a single

comprehensive online platform. LIMBS is a complete solution to court cases by entering the details about

the court case, it‘s progress, status, next date of hearing along with MIS reports to have a concurrent

watch by higher echelons as well as by the dealing person. LIMBS has unique features of e-Document

vault and Group SMS to sensitize the users. Advocate and Arbitration modules make this application

complete as all the stakeholders are brought at a single platform.

o Hence, only pair 1 is correctly matched.

https://telegram.me/UPSCMaterials https://telegram.me/FreeUPSCMaterials https://telegram.me/MaterialforExam

Page 32: VISION IAS - WordPress.com · 2018-11-05 · 3 ©Vision IAS Service Delivery Preparedness and achievement of Results Sound Public Grievance Redress Mechanism o Sevottam has also been

32 www.visionias.in ©Vision IAS

Q 80.A

o The Second Administrative Reforms Commission in its report entitled "Local Governance" has identified

the following core principles for effective local governance:

The principle of Subsidiarity: A central authority should have a subsidiary function, performing

only those tasks which cannot be performed at a more local level.

Effective democratic decentralization.

Proper delineation of functions: The roles and responsibilities of various tiers of government have

to be clearly defined.

Devolution in real terms: Devolution, to be real and meaningful, demands that local governments

should be effectively empowered to frame regulations, take decisions and enforce their will within

their legitimate sphere of action.

Convergence: In large, complex governance structures compartmentalization is inevitable. But as

governance is brought closer to the citizens, this fragmentation should yield place to convergence

based on the recognition that the citizens‘ needs and concerns are indivisible.

Citizen centricity: The citizen is the heart of a democratic system. Therefore all governance

institutions, particularly local governments should be judged by the satisfaction of citizens and the

direct empowerment of people.

Hence all the statements are correct.

Q 81.B

o Section 8(1) of the RTI Act lists down the exemption provisions. These exemptions can be used to deny

information sought by an applicant. These include:

Information, the disclosure of which would affect the sovereignty and integrity of India, the

security, strategic, scientific or economic interests of the State, relation with foreign State or

lead to incitement of an offense. Hence statement 2 is correct.

Information which has been expressly forbidden to be published by any court of law or tribunal or the

disclosure of which may constitute contempt of court.

Information, the disclosure of which would cause a breach of privilege of Parliament or the State

Legislature.

Information including commercial confidence, trade secrets or intellectual property.

Information available to a person in his fiduciary relationship. Hence statement 3 is orrect.

Information received in confidence from a foreign government.

Information, the disclosure of which would endanger the life or physical safety of any person or

identify the source of information or assistance given in confidence for law enforcement or security

purposes.

Information which would impede the process of investigation or apprehension or prosecution of

offenders;

Cabinet papers including records of deliberations of the Council of Ministers, Secretaries and other

officers: Provided that the decisions of Council of Ministers, the reasons for the decision and the

material on the basis of which the decisions were taken shall be made public after the decision has

been taken, and the matter is complete.

Information which relates to personal information the disclosure of which has no relationship to

any public activity or interest, or which would cause unwarranted invasion of the privacy of the

individual. Hence statement 4 is correct.

o The Act makes all of the exemptions contained in section 8(1) subject to a "Public Interest". What this

means is that even where requested information is covered by an exemption, the information should still

be disclosed to the applicant if the public interest in the specific case requires it.

o The Official secret Act 1923 is India's anti espionage ("Spy" and "Secret agent") act held over from

British colonisation. It states clearly that actions which involves helping an enemy state against India. It

also states that one cannot approach, inspect, or even pass over a prohibited government site or area.

https://telegram.me/UPSCMaterials https://telegram.me/FreeUPSCMaterials https://telegram.me/MaterialforExam

Page 33: VISION IAS - WordPress.com · 2018-11-05 · 3 ©Vision IAS Service Delivery Preparedness and achievement of Results Sound Public Grievance Redress Mechanism o Sevottam has also been

33 www.visionias.in ©Vision IAS

o The RTI Act, 2005 clearly says that in case of a clash with the OSA, the public interest will prevail.

Section 8(2) of the RTI Act says, “Notwithstanding anything in the Official Secrets Act, 1923, nor

any of the exemptions permissible in accordance with subsection 8(1) of RTI Act, a public authority

may allow access to information, if public interest in disclosure outweighs the harm to the protected

interests. Hence, only statement 1 is not correct.

Q 82.D

o The Union Government has constituted District Development Coordination and Monitoring Committee

(DISHA) for effective development, coordination of almost all the programmes of Central Government,

whether it is for infrastructure development or Social and human resource development.

o The main purpose of the committee is to coordinate with Central and State and local Panchayat

Governments, for successful and timely implementation of the schemes. The meetings of the committee

should be held once in every Quarter (Third Saturdays of April, July, October and February) and this has

been made mandatory. Hence, statement 1 is correct.

o The Chairperson of the committee is the senior most Member of Parliament (Lok Sabha) elected from the

district, nominated by the Ministry of Rural Development. The other Members of Parliament (Lok Sabha)

representing the district are designated as Co-Chairpersons. The Member Secretary of disha should be the

District Collector / District Magistrate/ Deputy Commissioner except in cases where a specific exemption

has been given by the Union Government. Hence, statement 2 and 3 are correct.

Q 83.D

o There are four types of interactions in e-Governance viz., government to citizens (G2C), government to

business (G2B), government to government, that is, inter-agency relationships (G2G), and government to

employees (G2E).

o Pair 1 is correctly matched: Shram Suvidha Portal facilitates businessmen to get all kinds of

registrations and submit returns that are required under labour laws at a single online window. It also

makes available to them the inspection reports prepared by the enforcement agency inspectors

online. Since it connects the government with businesses, it is an example of G2B interaction.

o Pair 2 is correctly matched: e-Office is a mission mode e-Governance project which paves for an

efficient and Open Government with a suite of products to go paperless and streamline workflows for both

intra and inter-government processes. Since it involves the interaction of government with the

government, it is an example of G2G interaction.

o Pair 3 is correctly matched: Aadhaar is a 12-digit unique identity number that can be obtained by

residents of India, based on their biometric and demographic data. It is being used for identifying the

beneficiaries for giving subsidies and other government benefits to the citizens. Since it involves the

interaction of government with citizens, it is an example of G2C interaction.

Q 84.D

o Statement 1 is not correct: Based on the recommendations of a Group of Secretaries made to the Prime

Minister, the Government decided that a one-stop Government e-Marketplace (GeM) would be created

to facilitate online procurement of common use Goods & Services required by various Government

Departments / Organizations / PSUs.

o It was launched on 9th August 2016, with the objective of creating an open, transparent and efficient

procurement platform for government.

o Statement 2 is not correct: More than 40% of transactions by volume are done with MSMEs registered

on the platform.

o It provides the tools of e-bidding, reverse e-auction and demand aggregation to facilitate the government

users achieve the best value for their money.

o The purchases through GeM by Government users have been authorized and made mandatory by Ministry

of Finance by adding a new Rule No. 149 in the General Financial Rules, 2017.

https://telegram.me/UPSCMaterials https://telegram.me/FreeUPSCMaterials https://telegram.me/MaterialforExam

Page 34: VISION IAS - WordPress.com · 2018-11-05 · 3 ©Vision IAS Service Delivery Preparedness and achievement of Results Sound Public Grievance Redress Mechanism o Sevottam has also been

34 www.visionias.in ©Vision IAS

Q 85.D

o The 74th Constitutional Amendment Act of 1992 bars the interference by courts in the electoral matters

of municipalities. It declares that the validity of any law relating to the delimitation of constituencies or

the allotment of seats to such constituencies cannot be questioned in any court. It further lays down that

no election to any municipality is to be questioned except by an election petition presented to such

authority and in such manner as provided by the state legislature. Hence option (d) is correct.

o The 73rd Constitutional Amendment Act of 1992 contains similar provisions in relation to the panchayats.

Q 86.B

o Hybrid Annuity Model (HAM) is one of the models used for the execution of PPP projects in India.

o HAM is a hybrid between EPC (engineering, procurement, and construction) and BOT (build, operate,

transfer) models. On behalf of the government, the agency releases 40 percent of the total project cost. It

is given in tranches linked to milestones. The balance 60 percent is arranged by the developer. Here, the

developer usually invests not more than 20-25 percent of the project cost while the remaining is raised as

debt. Hence, statement 1 is not correct

o HAM is a good trade-off, spreading the risk between developers and the Government. The private player

is responsible for the construction of the project. He is, however, not responsible for its ownership or

maintenance (which is done by the government). Hence, statement 2 is correct.

o It has primarily been used for the construction of highways. Hybrid annuity model (HAM) projects are

expected to constitute 60 percent of the projects awarded by the National Highway Authority of India

(NHAI) going forward. As per a Moody‘s report, the introduction of the hybrid annuity model (HAM) in

2016— as a variation of PPPs—has triggered new investment inflows. Hence, statement 3 is correct.

Q 87.C

o During his visit to Rwanda on 24th July, the Prime Minister participated in an event in Rweru Model

village outside Kigali under „Girinka programme‟, a social protection scheme of one cow - one

family and gifted cows as a contribution from India to the Scheme. India gifted US $200, 000 to

Rwanda to support the Girinka Programme of Government of Rwanda.

o About Girinka Programme:

It is the Rwanda‘s one of a kind Social Protection scheme under which the poorest residing in the

region get cows from the government and gift the first female calf to a neighbour to promote

brotherhood.

It aims to transform livelihoods, reconcile communities, and improve agricultural productivity

through the use of manure as fertilizers.

It has led to an increase in agricultural production in Rwanda - especially milk production and

products, reduced malnutrition and increased incomes.

India gifted 200 locally bought cows to villagers from Rweru village for Girinka.

Q 88.D

o The Constitution authorises the Parliament to determine the qualifications of members of the commission

and the manner in which they should be selected. Accordingly, the Parliament has specified the

qualifications of the chairman and members of the commission vide the Finance Commission Act,

1951. Hence, statement 1 is not correct.

o It is constituted by the President of India every fifth year or at such earlier time as he considers

necessary. Hence, statement 2 is not correct.

Q 89.C

o Statement 1 is not correct: e-Kranti is the second version of the National e-Governance Plan launched

by the government in 2015 with the vision of ―Transforming e-Governance for Transforming

Governance‖. The first version of the National e-Governance plan was launched in 2006. There are 44

https://telegram.me/UPSCMaterials https://telegram.me/FreeUPSCMaterials https://telegram.me/MaterialforExam

Page 35: VISION IAS - WordPress.com · 2018-11-05 · 3 ©Vision IAS Service Delivery Preparedness and achievement of Results Sound Public Grievance Redress Mechanism o Sevottam has also been

35 www.visionias.in ©Vision IAS

Mission Mode Projects under the e-Kranti programme which are grouped into Central, State, and

Integrated projects.

o Statement 2 is correct: Digital India is the flagship project of government which is based on three thrust

areas of infrastructure as a utility to every citizen, governance and services on demand and digital

empowerment of citizens. It is based on nine pillars - Broadband Highways Universal Access to Mobile

Connectivity; Public Internet Access Programme; e-Governance: Reforming Government through

Technology; e-Kranti - Electronic Delivery of Services; Information for All; Electronics

Manufacturing; IT for Jobs and Early Harvest Programmes.

o Statement 3 is correct: The key principles of e-Kranti are as follows:

Integrated Services and not Individual Services.

Government Process Reengineering (GPR) to be mandatory in every MMP.

ICT Infrastructure on Demand.

Cloud by Default.

Mobile First.

Mandating Standards and Protocols.

Language Localization.

Security and Electronic Data Preservation.

Q 90.C

o ‗Master of the Roster‘ refers to the privilege of the Chief Justice to constitute Benches to hear cases.

o This privilege was emphasized in November last year, when a Constitution Bench, declared that

“the Chief Justice is the master of the roster and he alone has the prerogative to constitute the

Benches of the Court and allocate cases to the Benches so constituted.

o This power had come under much public scrutiny after the four seniormost judges went public with their

grievances over the way sensitive political cases were being assigned to ‗junior judges‘. Several PILs

raising the matter of whether the CJI could still deal with a case when the allegations were against

himself.

o In case of appointments, the CJI in consultation with his other four senior colleagues recommends

appointments to the superior courts. A two-judge bench of the Supreme Court, however, refused to extend

this analogy to the realm of court administration. Instead, the bench chose to back the CJI‘s discretionary

powers to assign cases to benches.

o The court insisted that various factors go into the CJI assigning cases – these include specialization of

judges and their tenure in office. Any insistence that certain cases be assigned only to senior judges would

create a misconception about the ability of other top court judges to deal with all cases.

Q 91.A

o Airports Authority of India (AAI) was constituted by an Act of Parliament and came into being on

1st April 1995 by merging erstwhile National Airports Authority and International Airports Authority of

India. Hence statement 1 is correct.

o The merger brought into existence a single Organization entrusted with the responsibility of creating,

upgrading, maintaining and managing civil aviation infrastructure both on the ground and airspace in the

country.

o The functions of AAI are as follows:

Design, Development, Operation and Maintenance of international and domestic airports and

civil enclaves.

Control and Management of the Indian airspace extending beyond the territorial limits of the country,

as accepted by ICAO.

Construction, Modification and Management of passenger terminals.

Development and Management of cargo terminals at international and domestic airports.

Provision of passenger facilities and information system at the passenger terminals at airports.

https://telegram.me/UPSCMaterials https://telegram.me/FreeUPSCMaterials https://telegram.me/MaterialforExam

Page 36: VISION IAS - WordPress.com · 2018-11-05 · 3 ©Vision IAS Service Delivery Preparedness and achievement of Results Sound Public Grievance Redress Mechanism o Sevottam has also been

36 www.visionias.in ©Vision IAS

Expansion and strengthening of operation area, viz. Runways, Aprons, Taxiway etc.

Provision of visual aids.

Provision of Communication and Navigation aids.

o Safety aspects of civil aviation in India are governed by Directorate General of Civil Aviation

(DGCA). Hence statement 2 is not correct.

o About DGCA

It is an attached office of the Ministry of Civil Aviation.

The Directorate General of Civil Aviation is the regulatory body in the field of Civil Aviation

primarily dealing with safety issues. It is responsible for regulation of air transport

services to/from/within India and for enforcement of civil air regulations, air safety and airworthiness

standards. It also coordinates all regulatory functions with the International Civil Aviation

Organisation.

The headquarters are located in New Delhi with regional offices in the various parts of India

Tariffs for Air transport services are determined by Airports Economic Regulatory Authority

(AERA). Hence statement 3 is not correct.

About AERA: It is a statutory body constituted under the Airports Economic Regulatory Authority of

India Act, 2008 with its head office at Delhi. The statutory functions of the AERA as enshrined in the

Airports Economic Regulatory Authority of India Act, 2008 are as below:

o To determine the tariff for the aeronautical services.

o To determine the amount of the Development Fees in respect of major airports.

o To determine the amount of the Passengers Service Fee levied under Aircraft Rules, 1937 made under the

Aircraft Act, 1934.

o To monitor the set Performance Standards relating to quality, continuity and reliability of service as may

be specified by the Central Government or any authority authorized by it in this behalf.

o To perform such other functions relating to tariff, as may be entrusted to it by the Central Government or

as may be necessary to carry out the provisions of this Act.

Q 92.A

o Statement 1 is correct: Self-Help Group (SHG) is a small voluntary association of poor people,

preferably from the same socio-economic background. They come together for the purpose of solving

their common problems through self-help and mutual help. The SHG promotes small savings among its

members. The savings are kept with a bank. This common fund is in the name of the SHG. SHG is a

group formed by the community, which has a specific number of members like 15 or 20. Usually, the

number of members in one SHG does not exceed twenty. In such a group the poorest would come

together for emergency, disaster, social reasons, economic support to each other have ease of

conversation, social interaction and economic interactions.

o Statement 2 is not correct: An SHG can be an all-women group, all-men group, or even a mixed Group.

However, it has been the experience that women's groups perform better in all the important activities of

SHGs.

o Statement 3 is not correct: SHG is an informal group and registration under any Societies Act, State

cooperative Act or a partnership firm is not mandatory.

Q 93.C

o The Ribeiro Committee was set up in 1998 on the order of the Supreme Court following a Public Interest

Litigation (PIL) on police reforms. The committee proposed five major recommendations related to state

security, selection of DGP and complaints against the police, the recommendations have not been

implemented.

o The Committee on Civil Services Reforms, headed by P.C. Hota, former Chairman of the Union

Public Service Commission (UPSC), made a number of recommendations to improve the functioning of

https://telegram.me/UPSCMaterials https://telegram.me/FreeUPSCMaterials https://telegram.me/MaterialforExam

Page 37: VISION IAS - WordPress.com · 2018-11-05 · 3 ©Vision IAS Service Delivery Preparedness and achievement of Results Sound Public Grievance Redress Mechanism o Sevottam has also been

37 www.visionias.in ©Vision IAS

the all-India services, and to curb the propensity that a distressingly large number of the members of these

services show, to line their pockets by taking bribes and other dishonest means.

o In 2000, the then NDA government formed a panel headed by the former Chief Justice of Kerala and

Karnataka, Justice V.S. Malimath, to suggest an overhaul of the century-old criminal justice

system. Two years later, the Justice Malimath Committee submitted a report with 158 recommendations

to the Deputy Prime Minister, L.K. Advani, who was also the Home Minister. The Committee felt that the

existing system ―weighed in favour of the accused and did not adequately focus on justice to the victims

of crime.‖

o The Indrajit Gupta Committee (1998) endorsed state funding of elections, seeing ―full justification

constitutional, legal as well as on ground of public interest‖ in order to establish a fair playing field for

parties with less money. The Committee recommended two limitations to state funding. Firstly, that state

funds should be given only to national and state parties allotted a symbol and not to independent

candidates. Secondly, that in the short-term state funding should only be given in kind, in the form of

certain facilities to the recognized political parties and their candidates. The Committee noted that at the

time of the report the economic situation of the country only suited partial and not full state funding of

elections.

o Hence pairs 2 and 3 are correctly matched.

Q 94.A

o The Multilateral Instrument (MLI) and its Explanatory Statement were developed through a

negotiation involving more than 100 countries and jurisdictions and adopted on 24 November 2016, under

a mandate delivered by G20 Finance Ministers and Central Bank Governors at their February 2015

meeting. The Multilateral Convention to Implement Tax Treaty Related Measures to Prevent Base

Erosion and Profit Shifting ("Multilateral Instrument" or "MLI") that will swiftly implement a series of

tax treaty measures to update international tax rules and lessen the opportunity for tax avoidance by

multinational enterprises.

o At a meeting of G20 finance ministers and central bankers held at Argentina in July 2018, India pitched

for a global mechanism technically called Multilateral Instrument (MLI) which would be a permanent

measure to tax digital companies that earn revenues from a large user base in the country. This would

automatically amend bilateral tax treaties to include the taxation provisions for digital business.

o In Budget 2015-16, a 6% equalization levy was introduced to bring to tax payments made for online

advertisement services.

o BEPS refers to tax planning strategies that exploit gaps and mismatches in tax rules to artificially shift

profits to low or no-tax locations where there is little or no economic activity.

Q 95.A

o Statement 1 is not correct: Central Ground Water Board (CGWB), a subordinate office of the Ministry

of Water Resources, Government of India.

o Statement 2 is correct: It is entrusted with the responsibilities of providing scientific inputs for

management, exploration, monitoring, assessment, augmentation and regulation of ground water resources

of the country. Major activities being taken up by Central Ground Water Board include macro/micro-level

groundwater management studies, exploratory drilling programme, monitoring of groundwater levels and

water quality through a network of groundwater observation wells comprising both large diameter open

wells and purpose-built bore/tube wells (piezometers), implementation of demonstrative schemes for

artificial recharge and rainwater harvesting for recharge augmentation

o Statement 3 is not correct: Central Ground Water Board (CGWB) is constituted under the Environment

(Protection) Act, 1986. Central Ground Water Board also provides technical know-how for scientific

ground water exploration, development and management to various stakeholders.

https://telegram.me/UPSCMaterials https://telegram.me/FreeUPSCMaterials https://telegram.me/MaterialforExam

Page 38: VISION IAS - WordPress.com · 2018-11-05 · 3 ©Vision IAS Service Delivery Preparedness and achievement of Results Sound Public Grievance Redress Mechanism o Sevottam has also been

38 www.visionias.in ©Vision IAS

Q 96.C

o The Preamble of the Constitution of India provides the moral compass for our country‟s

governance structure.

o The values of celebrating diversity, respect for individuals and different sections of the population and the

resolve to ensure social, economic and political justice, liberty of thought, expression, belief, faith and

worship, equity of status and opportunity and promotion of fraternity assuring the dignity of the individual

form the cornerstone of our democratic, republic which took shape 70 years ago.

o The legislature, the executive and the judiciary have necessarily been turning to the Preamble for

guidance. The actions of the three wings of our polity are judged by the extent to which they deepen the

roots of our democratic framework.

Q 97.B

o The Petroleum and Natural Gas Regulatory Board (PNGRB) was constituted under the provisions of the

Petroleum & Natural Gas Regulatory Board Act, 2006. The board has also been mandated to regulate

the refining, processing, storage, transportation, distribution, marketing and sale of petroleum,

petroleum products and natural gas excluding production of crude oil and natural gas so as and to

ensure uninterrupted and adequate supply of petroleum, petroleum products and natural gas in all

parts of the country. Hence statement 1 is not correct.

o The Board shall-

Protect the interest of consumers by fostering fair trade and competition amongst the

entities; Hence statement 3 is correct.

Register entities to-

market notified petroleum and petroleum products and, subject to the contractual obligations of

the Central Government, natural gas;

establish and operate liquefied natural gas terminals;

establish storage facilities for petroleum, petroleum products or natural gas exceeding such

capacity as may be specified by regulations;

o Authorise entities to-

lay, build, operate or expand a common carrier or contract carrier;

lay, build, operate or expand city or local natural gas distribution network;

o Declare pipelines as common carrier or contract carrier;

o Regulate, by regulations,-

access to common carrier or contract carrier so as to ensure fair trade and competition amongst

entities and for that purpose specify pipeline access code;

transportation rates for common carrier or contract carrier;

access to city or local natural gas distribution network so as to ensure fair trade and competition

amongst entities as per pipeline access code;

o In respect of notified petroleum, petroleum products and natural gas-

ensure adequate availability;

ensure display of information about the maximum retail prices fixed by the entity for consumers at

retail outlets;

monitor prices and take corrective measures to prevent restrictive trade practice by the entities;

secure equitable distribution for petroleum and petroleum products;

provide, by regulations, and enforce, retail service obligations for retail outlets and marketing service

obligations for entities;

monitor transportation rates and take corrective action to prevent restrictive trade practice by the

entities;

o It doesnot determine the prices of petroleum products in India. Hence statement 2 is not correct.

https://telegram.me/UPSCMaterials https://telegram.me/FreeUPSCMaterials https://telegram.me/MaterialforExam

Page 39: VISION IAS - WordPress.com · 2018-11-05 · 3 ©Vision IAS Service Delivery Preparedness and achievement of Results Sound Public Grievance Redress Mechanism o Sevottam has also been

39 www.visionias.in ©Vision IAS

Q 98.A

o The Parliament has passed Constitution (123rd Amendment) Bill, 2017 that grants the National

Commission on Backward Classes (NCBC) constitutional status, at par with the National Commission for

Scheduled Castes (NCSC) and the National Commission for Scheduled Tribes. NCBC is now a

Constitutional body under Article 338B of the Indian Constitution. Hence statement 1 is correct.

o Under the Constitution Amendment Bill, the NCBC will comprise of five members appointed by the

President. Their tenure and conditions of service will also be decided by the President. The duties of the

NCBC will include:

Investigating and monitoring how safeguards provided to the backward classes under the Constitution

and other laws are being implemented.

Inquiring into specific complaints regarding violation of rights. Hence statement 2 is correct.

Advising and making recommendations on the socio-economic development of such classes.

o The Constitution Amendment Bill states that the President may specify the socially and educationally

backward classes in the various states and union territories. He may do this in consultation with the

Governor of the concerned state. However, a law of Parliament will be required if the list of backward

classes is to be amended. Hence statement 3 is not correct.

Q 99.D

o Regulation is an attempt to control or influence private behavior in the desired direction by imposing costs

on or proscribing undesirable behavior.

o The rationale for Regulation: Regulation can have important consequences for economic efficiency and

private incentives, it is usually justified only under special conditions. Accordingly, there are three sets of

justifications for regulatory interventions:

o Prevention of market failures - Market failure is a condition in which the market mechanism fails to

allocate resources efficiently to maximize social welfare. Market failures occur in the provision of public

goods, in the case of natural monopolies or asymmetric information, and in the presence of externalities.

o Restriction or removal of anti-competitive practices - Firms may resort to anti-competitive practices

such as price fixing, market sharing or abuse of dominant or monopoly power. Laws that empower

officials to take action can help deter such practices. Regulation through a set of transparent, consistent,

and non-discriminatory rules can create a competitive and dynamic environment in which market players

can thrive. In its absence, anticompetitive practices and regulatory failures may not allow the market

process to yield socially optimal outcomes.

o Promotion of public interest - Ensuring fair access, non-discrimination, affirmative action, or any other

matter of public importance provide an important reason for regulation.

o At times public interest can be manipulated by lobbies to further their vested interests.

o Hence, all the statements are correct.

Q 100.B

o What is an Early Harvest Scheme/Programme (EHS)? Early harvest scheme is a precursor to a free

trade agreement (FTA) between two trading partners. This is to help the two trading countries to

identify certain products for tariff liberalization pending the conclusion of FTA negotiation.

o It is primarily a confidence-building measure. A good example of an EHS is between India and Thailand

signed in October 2003, wherein 83 products were identified to be reduced to zero in a phased manner.

The EHS has been used as a mechanism to build greater confidence amongst trading partners to prepare

them for even bigger economic engagement. It is also called as Early Harvest Package sometimes.

o India and South Korea signed a joint statement in July 2018 agreeing to discuss an ‗early harvest‘ package

for the Comprehensive Economic Partnership Agreement (CEPA) aimed at identifying key areas for

enhanced trade between the two countries.

Copyright © by Vision IAS

All rights are reserved. No part of this document may be reproduced, stored in a retrieval system or transmitted

in any form or by any means, electronic, mechanical, photocopying, recording or otherwise, without prior

permission of Vision IAS

https://telegram.me/UPSCMaterials https://telegram.me/FreeUPSCMaterials https://telegram.me/MaterialforExam